New classes are starting soon - secure your spot today!

2017 AMC 10/12 A Discussion

Go back to the Math Jam Archive

AoPS Instructors will discuss problems from the AMC 10/12 A, administered February 7. We will discuss the last 5 problems on each test.

Copyright © 2024 AoPS Incorporated. This page is copyrighted material. You can view and print this page for your own use, but you cannot share the contents of this file with others.

Facilitator: AoPS Staff

devenware 2017-02-08 19:01:35
Welcome to the 2017 AMC 10A/12A Math Jam!
devenware 2017-02-08 19:01:42
I'm Deven Ware, and I'll be leading our discussion tonight.
devenware 2017-02-08 19:01:49
I've worked at AoPS since 2014. I left the Shire at an early age and honed my lethal spellcasting skills under the watchful guidance of some of the most famed wizards of Middle Earth. My hobbies include freestyle hopscotch, competitive jazz viola, and extreme grasshopper racing. I have a teenaged pet bonsai tree named Shigeru.
VGoma 2017-02-08 19:01:59
How does this work?
devenware 2017-02-08 19:02:02
Before we get started I would like to take a moment to explain our virtual classroom procedures to those who have not previously participated in a Math Jam or one of our online classes.
devenware 2017-02-08 19:02:10
The classroom is moderated, meaning that students can type into the classroom, but these comments will not go directly into the room. These comments go to the instructors, who may choose to share your comments with the room.
devenware 2017-02-08 19:02:17
This helps keep the class organized and on track. This also means that only well-written comments will be dropped into the classroom, so please take time writing responses that are complete and easy to read.
devenware 2017-02-08 19:02:30
There are bunches and bunches of students here. As I said, only a fraction of the well-written comments will be passed to the entire group. Please do not take it personally if your comments do not get posted, and please do not complain about it. I expect this Math Jam to be much larger than our typical class, so please be patient with me---there are quite a few of you here tonight!!
devenware 2017-02-08 19:02:39
Also, we won't be going through the math quite as thoroughly as we do in our classes -- I can't teach all the prerequisite material for every problem as we go. Another difference between tonight and our regular online classes is that it is very unlikely that we'll be able to answer every single question you ask. We usually do in our classes, but we have a large number of students tonight! So, please go ahead and ask questions, but also please understand if we aren't able to answer them all!
devenware 2017-02-08 19:03:05
Your assistants for today will be Kaitlin Maile (kmaile32) and Kevin Chong An (projeceulerlover).
devenware 2017-02-08 19:03:09
Kaitlin has been working for AOPS since November 2014, when she was a senior in high school. While in high school, Kaitlin participated in math team, marching band, and various sports. Now, she is a junior at the University of Minnesota, majoring in Biomedical Engineering and minoring in Computer Science. When not working for AOPS, studying for classes, or working on research projects, she enjoys weight-lifting, listening to almost any genre of music, and going on adventures.
devenware 2017-02-08 19:03:11
Kevin is currently a senior at California Institute of Technology majoring in Mathematics with a minor in Computer Science. He's done math competitions throughout his time at high school, and he really enjoys sharing his knowledge with others. Some of his non-problem solving interests are weightlifting, playing Hanabi, and video games.
devenware 2017-02-08 19:03:18
They will be sending you messages to answer questions or offer other help. However, due to the incredibly large size of the session tonight, they may not be able to get to you right away (or at all). Repeating your question over and over may give you warm feelings, but just makes their poor lives harder, so please, only ask your question once and be patient, and please understand that we may not be able to answer all the questions tonight.
devenware 2017-02-08 19:03:20
Please also remember that the purpose of this Math Jam is to work through the solutions to AMC problems and learn! "Working through the solutions" includes discussing problem-solving tactics. So please, when a question is posted, do not simply respond with the final answer (since I'll be ignoring those anyway). That's not what we're doing tonight. We're going to work through the problems step-by-step, and comments that skip key steps or jump ahead in the problem, without providing explanation or motivation, won't be posted.
AmitLuke 2017-02-08 19:03:26
How do you type so fast?
devenware 2017-02-08 19:03:35
I use several keyboards.
AmitLuke 2017-02-08 19:04:00
Cool
hodori01 2017-02-08 19:04:00
jacksonhu 2017-02-08 19:04:00
cool
Benjy450 2017-02-08 19:04:00
Impressive.
Wave-Particle 2017-02-08 19:04:00
padfoot6302 2017-02-08 19:04:00
interesting
IceParrot 2017-02-08 19:04:00
Really!?!
devenware 2017-02-08 19:04:05
devenware 2017-02-08 19:04:18
We will work the last 5 problems from the AMC 10A, then the last 5 problems from the AMC 12A. Two of these problems are the same, 10A Problem 24 and 12A Problem 23. We'll only solve that problem once.
devenware 2017-02-08 19:04:23
Let's get started!
devenware 2017-02-08 19:04:25
Oh, and there truly are a lot of people here tonight. I really don't like saying it, but we're probably going to miss some of the things that some of you say. (Especially during the geometry problems - sheesh.) Please forgive me in advance. That doesn't happen in our classes.
devenware 2017-02-08 19:04:41
OKAY. READY?
mathchampion1 2017-02-08 19:05:12
yes!
mj434 2017-02-08 19:05:12
YES!
wiler5002 2017-02-08 19:05:12
IM READY
XxkalleknightxXlelxd 2017-02-08 19:05:12
yes
mathaction 2017-02-08 19:05:12
YEAH
AmitLuke 2017-02-08 19:05:12
YES
vlwilliam 2017-02-08 19:05:12
GO!\
Auty 2017-02-08 19:05:12
YEAH!
B_Blake 2017-02-08 19:05:12
ya
mepehe888 2017-02-08 19:05:12
yup
padfoot6302 2017-02-08 19:05:12
yea1!!!
challengetogo 2017-02-08 19:05:12
READY!!
mathcrazymj 2017-02-08 19:05:12
YESH
ruchirk533 2017-02-08 19:05:12
yay
Smoothfang 2017-02-08 19:05:12
YES
Cardinals2014 2017-02-08 19:05:12
yeah
User2013 2017-02-08 19:05:12
yes!
PenguinJoe 2017-02-08 19:05:12
YES
iks92 2017-02-08 19:05:12
yeah!!
GeronimoStilton 2017-02-08 19:05:12
Ready!
AmitLuke 2017-02-08 19:05:12
IM READY
bluephoenix 2017-02-08 19:05:12
YEAH!!!!
JTMath 2017-02-08 19:05:12
YES
kiwitrader123 2017-02-08 19:05:12
Yea
Bill9000 2017-02-08 19:05:12
YES! I'm excited!
devenware 2017-02-08 19:05:18
OKAY, LET'S GO!
devenware 2017-02-08 19:05:24
21. A square with side length $x$ is inscribed in a right triangle with sides of length 3, 4, and 5 so that one vertex of the square coincides with the right-angle vertex of the triangle. A square with side length $y$ is inscribed in another right triangle with sides 3, 4, and 5 so that one side of the square lies on the hypotenuse of the triangle. What is $\dfrac{x}{y}$?
$\phantom{hi}$
$\text{(A) } \dfrac{12}{13} \quad
\text{(B) } \dfrac{35}{37} \quad
\text{(C) } 1 \quad
\text{(D) } \dfrac{37}{35} \quad
\text{(E) } \dfrac{13}{12}$
devenware 2017-02-08 19:05:32
What's the first step?
GeneralCobra19 2017-02-08 19:05:52
Draw a diagram
hodori01 2017-02-08 19:05:52
draw diagram
quartzgirl 2017-02-08 19:05:52
First, draw a diagram
Talker32332 2017-02-08 19:05:52
Diagram!
zjjc123 2017-02-08 19:05:52
diagram
mathforfun2016 2017-02-08 19:05:52
Draw it.
mt492 2017-02-08 19:05:52
diagram
Jessp 2017-02-08 19:05:52
draw a diagram
mikhailgromov 2017-02-08 19:05:52
diagram
Brisingrfire 2017-02-08 19:05:52
draw a picture
theironcatfish 2017-02-08 19:05:52
Draw it out?
uptownmath 2017-02-08 19:05:52
Draw a diagram.
SomethingNeutral 2017-02-08 19:05:52
Draw a diagram
garretth 2017-02-08 19:05:52
Draw the diagram
devenware 2017-02-08 19:06:06
Draw a diagram. That's always my first step on a geometry problem.
devenware 2017-02-08 19:06:11
pie314159265 2017-02-08 19:06:30
use similar triangles
S0larPh03nix 2017-02-08 19:06:30
similar triangles
Liopleurodon 2017-02-08 19:06:30
Similar triangles!
devenware 2017-02-08 19:06:34
And what does similar triangles tell us?
devenware 2017-02-08 19:07:43
Lots of great answers out there, I'm going with the one that I did when solving this:
Picroft 2017-02-08 19:07:49
(4-x)/x = 4/3
amc8nov 2017-02-08 19:07:49
(4-x)/x=4/3
dragon6688 2017-02-08 19:07:49
$\frac{4-x}{x} = \frac{4}{3}$
devenware 2017-02-08 19:07:58
The lower left triangle is a 3-4-5 triangle so\[\frac{4-x}x=\frac43.\]
devenware 2017-02-08 19:08:00
Therefore\[12-3x=4x,\]so $x=\dfrac{12}7$.
devenware 2017-02-08 19:08:04
Now what?
jybsmartguy2 2017-02-08 19:08:33
draw another diagram for y
CrystalEye 2017-02-08 19:08:33
find y
liant 2017-02-08 19:08:33
find y
mathaction 2017-02-08 19:08:33
draw the other?
acegikmoqsuwy2000 2017-02-08 19:08:33
find $y$
dudethefirst 2017-02-08 19:08:33
find y
GeronimoStilton 2017-02-08 19:08:33
Solve for $y$.
Hyun04 2017-02-08 19:08:33
draw another diagram
tdeng 2017-02-08 19:08:33
Now draw the other square
mathforfun2016 2017-02-08 19:08:33
Draw diagram for y.
kiwitrader123 2017-02-08 19:08:33
draw the other diagram
skmc 2017-02-08 19:08:33
Draw the other diagram
MathMan1234 2017-02-08 19:08:33
draw another diagram for y
mikhailgromov 2017-02-08 19:08:33
diagram for y
CrystalEye 2017-02-08 19:08:33
draw a diagram to find y
oriduck.nk 2017-02-08 19:08:33
draw another diagram
blacksheep2003 2017-02-08 19:08:33
Draw the second triangle, and mark the diagram
Ani10 2017-02-08 19:08:33
find y using another diagram
devenware 2017-02-08 19:08:37
The other diagram:
devenware 2017-02-08 19:08:39
devenware 2017-02-08 19:08:50
(The 3 and 4 are the long edge lengths.)
devenware 2017-02-08 19:08:56
Let's get some more numbers on here. What are these two values $a$ and $b$?
devenware 2017-02-08 19:08:56
vlwilliam 2017-02-08 19:09:22
a=5-y-b
quanhui868 2017-02-08 19:09:22
5-y-b
yid 2017-02-08 19:09:22
a+y+b = 5
ruchirk533 2017-02-08 19:09:22
a+b+y=5
devenware 2017-02-08 19:09:31
Definitely. We'll keep that in mind.
devenware 2017-02-08 19:09:39
Can we use similar triangles again to relate $a,b,$ and $y$?
CrystalEye 2017-02-08 19:10:17
a=4/3y b=3/4y
dudethefirst 2017-02-08 19:10:17
4/3y and 3/4y
acegikmoqsuwy2000 2017-02-08 19:10:17
$\dfrac 43y$ and $\dfrac 34y$
uptownmath 2017-02-08 19:10:17
a=4y/3, b=3y/4
Arieoreos 2017-02-08 19:10:17
a=4/3y, b=3/4y
summitwei 2017-02-08 19:10:17
a=4/3*y, b=3/4*y
brainiac1 2017-02-08 19:10:17
a=4y/3, b=3y/4
sydney_z 2017-02-08 19:10:17
a = 4y/3
BunBulb 2017-02-08 19:10:17
a/y = 4/3
sydney_z 2017-02-08 19:10:17
b=3y/4
devenware 2017-02-08 19:10:22
Everything is a 3-4-5 triangle.
devenware 2017-02-08 19:10:23
The left triangle gives $\dfrac{a}y=\dfrac43$, so $a=\dfrac{4y}3$.
devenware 2017-02-08 19:10:25
The right triangle gives $\dfrac{b}y=\dfrac34$, so $b=\dfrac{3y}4$.
devenware 2017-02-08 19:10:26
devenware 2017-02-08 19:10:38
Okay, now what?
amwmath 2017-02-08 19:11:07
Sum of the things on the bottom is 5
tdeng 2017-02-08 19:11:07
Now 4y/3 + y + 3y/4=5
MathMan1234 2017-02-08 19:11:07
now 4y/3 + 3y/4 + y = 5
yid 2017-02-08 19:11:07
a+b+y = 5
wiler5002 2017-02-08 19:11:07
$\frac{4y}{3}+y+\frac{3y}{4}=5$
CrystalEye 2017-02-08 19:11:07
4/3y+y+3/4y=5
dragon6688 2017-02-08 19:11:07
$\frac{4y}{3} + y + \frac{3y}{4} = 5$
ScienceSpirit 2017-02-08 19:11:07
Equate to 5
devenware 2017-02-08 19:11:20
Now we use the fact that everything on the bottom adds to 5!
devenware 2017-02-08 19:11:24
What do you get for $y$?
mj434 2017-02-08 19:12:08
y=60/37
blacksheep2003 2017-02-08 19:12:08
$y=60/37$
gmdoss 2017-02-08 19:12:08
60/37
n2001 2017-02-08 19:12:08
60/37
mathusername 2017-02-08 19:12:08
y=60/37
sunnywoods 2017-02-08 19:12:08
y=60/37
Cardinals2014 2017-02-08 19:12:08
60/37
CharlesHong 2017-02-08 19:12:08
60/37
AAANNNMMMIII 2017-02-08 19:12:08
60/37
tfz4629 2017-02-08 19:12:08
60/37
CoolVincent122333 2017-02-08 19:12:08
60/37
evanhlu 2017-02-08 19:12:08
60/37
devenware 2017-02-08 19:12:20
The bottom edge has length 5, so we get the equation\[5=\frac{3y}4+y+\frac{4y}3=\frac{9y+12y+16y}{12}=\frac{37y}{12}.\] Therefore $y=\dfrac{60}{37}$.
devenware 2017-02-08 19:12:21
And the answer?
theartof 2017-02-08 19:12:54
D
MSTang 2017-02-08 19:12:54
D, 37/35
Mathisfun04 2017-02-08 19:12:54
37/35
Wave-Particle 2017-02-08 19:12:54
D
bobjoe123 2017-02-08 19:12:54
So x/y = 37/35
garretth 2017-02-08 19:12:54
D
tooswagforyou 2017-02-08 19:12:54
37/35
uptownmath 2017-02-08 19:12:54
37/35
mikhailgromov 2017-02-08 19:12:54
D
Cardinals2014 2017-02-08 19:12:54
D
zhengyf 2017-02-08 19:12:54
D) 37/35
FATRaichu 2017-02-08 19:12:54
37/35
AmitLuke 2017-02-08 19:12:54
D
Mario2357 2017-02-08 19:12:54
D) 37/35
padfoot6302 2017-02-08 19:12:54
37/35
SomethingNeutral 2017-02-08 19:12:54
37/35
conniejeon1 2017-02-08 19:12:54
37/35
Liopleurodon 2017-02-08 19:12:54
$\frac{37}{35}$
devenware 2017-02-08 19:13:00
We want \[\dfrac xy=\frac{\frac{12}7}{\frac{60}{37}}=\frac{12\cdot37}{60\cdot7}=\boxed{\dfrac{37}{35}}.\]
devenware 2017-02-08 19:13:05
The answer is (D).
dragon6688 2017-02-08 19:13:35
Makes it look so easy...
theironcatfish 2017-02-08 19:13:35
Well thats pretty neat
SmartGuy101 2017-02-08 19:13:35
YAY
Talker32332 2017-02-08 19:13:35
easier than i thought
Brainiac2 2017-02-08 19:13:35
Yay!
devenware 2017-02-08 19:13:40
devenware 2017-02-08 19:13:45
NEXT PROBLEM.
devenware 2017-02-08 19:14:00
22. Sides $\overline{AB}$ and $\overline{AC}$ of equilateral triangle $ABC$ are tangent to a circle at points $B$ and $C$, respectively. What fraction of the area of $\triangle ABC$ lies outside the circle?
$\phantom{hi}$
$\text{(A) } \dfrac{4\sqrt{3}\pi}{27} - \dfrac13\quad
\text{(B) } \dfrac{\sqrt{3}}{2} - \dfrac{\pi}8\quad
\text{(C) } \dfrac12 \quad
\text{(D) } \sqrt{3} - \dfrac{2\sqrt{3}\pi}{9} \quad
\text{(E) } \dfrac43-\dfrac{4\sqrt{3}\pi}{27}$
devenware 2017-02-08 19:14:07
Okay, everyone better know the first step...
theartof 2017-02-08 19:14:41
Draw a diagram
quartzgirl 2017-02-08 19:14:41
Again, draw a diagram.
Wave-Particle 2017-02-08 19:14:41
again draw a diagram
SomethingNeutral 2017-02-08 19:14:41
Draw another diagram
rplamkin 2017-02-08 19:14:41
draw a diagram
N3RDBIRD 2017-02-08 19:14:41
draw
mathchampion1 2017-02-08 19:14:41
diagram
chessapple9 2017-02-08 19:14:41
Draw a diagram
ShineBunny 2017-02-08 19:14:41
Diagram!
sbundlab1 2017-02-08 19:14:41
Diagram!
mathcrazymj 2017-02-08 19:14:41
draw diagram
Goliath 2017-02-08 19:14:41
diagram
jeffshen 2017-02-08 19:14:41
draw it
purpleapples 2017-02-08 19:14:41
draw a diagram
TaffyQ 2017-02-08 19:14:41
draw a diagram!
challengetogo 2017-02-08 19:14:41
diagram!
Yuny 2017-02-08 19:14:41
diagramm
thinmint 2017-02-08 19:14:41
Draw a picture!!!
GM2B 2017-02-08 19:14:41
diagram
Auty 2017-02-08 19:14:41
Draw a diagram!
Happy2020 2017-02-08 19:14:41
draw a diagram!
dzhou100 2017-02-08 19:14:41
DRAW A PIC/DIAGRAM!
GeneralCobra19 2017-02-08 19:14:41
DIAGRAm
kevindk 2017-02-08 19:14:41
diagram
PenguinJoe 2017-02-08 19:14:41
DIAGRAM
dipenm 2017-02-08 19:14:41
draw diagram
devenware 2017-02-08 19:14:47
DRAW A DIAGRAM!
devenware 2017-02-08 19:14:54
When sketching this diagram, where should you start?
Arieoreos 2017-02-08 19:15:21
Draw the circle first
kiwitrader123 2017-02-08 19:15:21
Circle
jybsmartguy2 2017-02-08 19:15:21
the circle
WW92030 2017-02-08 19:15:21
circle
Mario2357 2017-02-08 19:15:21
the circle
amc8nov 2017-02-08 19:15:21
with the circle
rplamkin 2017-02-08 19:15:21
the circle
MSTang 2017-02-08 19:15:21
circle
wiler5002 2017-02-08 19:15:21
With the circle
mathchampion1 2017-02-08 19:15:21
the circle
theironcatfish 2017-02-08 19:15:21
circle then triangle
mathforfun2016 2017-02-08 19:15:21
drawing the circle
devenware 2017-02-08 19:15:26
It's hard to draw a circle tangent to two lines. It's a lot easier to draw the circle then the tangent lines.
devenware 2017-02-08 19:15:29
devenware 2017-02-08 19:15:32
Anything else we should add to the diagram before we get started?
uptownmath 2017-02-08 19:16:10
radii
dipenm 2017-02-08 19:16:10
the center
wiler5002 2017-02-08 19:16:10
Center of the circle and the right angles it makes
purpleapples 2017-02-08 19:16:10
center of circle
dipenm 2017-02-08 19:16:10
connect the center to the tangent points
Arieoreos 2017-02-08 19:16:10
Two radii
theartof 2017-02-08 19:16:10
Radii
Liopleurodon 2017-02-08 19:16:10
The radii to the points of tangency
CrystalEye 2017-02-08 19:16:10
radii
orchdork123 2017-02-08 19:16:10
points
amc8nov 2017-02-08 19:16:10
draw radii
vbcnxm 2017-02-08 19:16:10
radii
abvenkgoo 2017-02-08 19:16:10
Radii from the points of tangency to the center
Wave-Particle 2017-02-08 19:16:10
radii of circle
devenware 2017-02-08 19:16:14
Whenever we have tangents to a circle, it makes sense to draw the perpendicular radii.
devenware 2017-02-08 19:16:15
devenware 2017-02-08 19:16:18
What regions should we label?
dipenm 2017-02-08 19:17:26
the circular segment
amc8nov 2017-02-08 19:17:26
the triangle outside of the circle
kiwitrader123 2017-02-08 19:17:26
region outside circle, sector
GeronimoStilton 2017-02-08 19:17:26
The triangle, the bit of the sector not including the triangle
Mario2357 2017-02-08 19:17:26
iscoceles triangle, missing arc, and other part of the equilateral
chessapple9 2017-02-08 19:17:26
We should label the region in the triangle outside of the circle and inside of the circle.
WW92030 2017-02-08 19:17:26
outside the circle, inside the circle
Liopleurodon 2017-02-08 19:17:26
The area of the triangle outside the circle, the area of the circle that's outside the triangle but inside the circle, and the part that they share
devenware 2017-02-08 19:17:30
Let's label the part of the triangle outside the circle, the part of the triangle inside the circle, and that other little triangle that just happened to appear.
devenware 2017-02-08 19:17:32
devenware 2017-02-08 19:17:33
And what value are we trying to compute?
devenware 2017-02-08 19:17:57
(READ THE PROBLEM CAREFULLY!)
happypi3.14159265358979 2017-02-08 19:18:46
x/(x+y)
Mathisfun04 2017-02-08 19:18:46
x/(x + y)
Happy2020 2017-02-08 19:18:46
x/(x+y)
wiler5002 2017-02-08 19:18:46
$\frac{x}{x+y}$
User2013 2017-02-08 19:18:46
x/(x+y)
aragornmf 2017-02-08 19:18:46
x/(x+y)
AAANNNMMMIII 2017-02-08 19:18:46
x/(x+y)
iks92 2017-02-08 19:18:46
x/(x+y)
BunBulb 2017-02-08 19:18:46
x/(x+y)
ScienceSpirit 2017-02-08 19:18:46
x/(x+y)
CrystalEye 2017-02-08 19:18:46
x/(x+y)
mt492 2017-02-08 19:18:46
x/(x+y)
blacksheep2003 2017-02-08 19:18:46
$x/(x+y)$
devenware 2017-02-08 19:18:50
We want to find $\dfrac{x}{x+y}$.
devenware 2017-02-08 19:18:52
Note that this doesn't have anything to do with the $z$ we labeled. So, even if we might use it to help, we'll want to get rid of it from our computations at the end.
devenware 2017-02-08 19:18:57
Before we get started, is there anything we can do to simplify our computations?
Arieoreos 2017-02-08 19:19:34
Make the circle a unit circke
amwmath 2017-02-08 19:19:34
Set the radius to $1$
warrenwangtennis 2017-02-08 19:19:34
r = 1
brainiac1 2017-02-08 19:19:34
assume a side length or radius or 1
challengetogo 2017-02-08 19:19:34
let the radius of the circle be 1
MSTang 2017-02-08 19:19:34
let the circle have radius 1
dragon6688 2017-02-08 19:19:34
Set the radius to 1 WLOG because the end result is a ratio
mathchampion1 2017-02-08 19:19:34
let the radius be 1
wiler5002 2017-02-08 19:19:34
Let the radius be 1
ScienceSpirit 2017-02-08 19:19:34
Make radius 1
devenware 2017-02-08 19:19:41
Nothing in the problem depends on how big our diagram is -- we can scale it up or down and it doesn't matter. So, we can just pick some length to be whatever we want.
devenware 2017-02-08 19:19:46
At first we might want the edge length of the equilateral triangle to be 1, but since the circle seems like it's going to be important in a couple of computations, let's set that to 1:
devenware 2017-02-08 19:19:48
devenware 2017-02-08 19:19:54
What is the edge-length of the equilateral triangle now?
MSTang 2017-02-08 19:20:32
$\sqrt{3}$
dudethefirst 2017-02-08 19:20:32
root 3
GeronimoStilton 2017-02-08 19:20:32
$\sqrt{3}$
WW92030 2017-02-08 19:20:32
root 3
CrystalEye 2017-02-08 19:20:32
sqrt3
chessapple9 2017-02-08 19:20:32
$\sqrt 3$
MSTang 2017-02-08 19:20:32
dipenm 2017-02-08 19:20:32
$\sqrt{3}$
throwhit 2017-02-08 19:20:32
sqrt(3)
mathchampion1 2017-02-08 19:20:32
sqrt(3)
fields123 2017-02-08 19:20:32
sqr3
cakeguy 2017-02-08 19:20:32
$\sqrt{3}$
purplecoat 2017-02-08 19:20:32
sqrt 3
AidanNReilly 2017-02-08 19:20:32
sqrt 3
adyj 2017-02-08 19:20:32
root 3
skmc 2017-02-08 19:20:32
sqrt(3)
devenware 2017-02-08 19:20:40
This triangle is a 30-60-90 triangle since the left vertex has measure $\dfrac{60^\circ}2$ and the top vertex is a right angle:
devenware 2017-02-08 19:20:41
devenware 2017-02-08 19:20:46
Therefore the equilateral triangle has edge-length $\sqrt3$:
devenware 2017-02-08 19:20:47
devenware 2017-02-08 19:21:08
How do $x,y,$ and $z$ relate to the triangles we've drawn and to the circle?
warrenwangtennis 2017-02-08 19:21:41
x+y=area of triangle
MSTang 2017-02-08 19:21:41
x+y is now the area of the triangle
conniejeon1 2017-02-08 19:21:41
x+y= Area of the eqilateral triangle
devenware 2017-02-08 19:21:47
We know that $x + y$ is the area of the equilateral triangle. So, \[x + y = (\sqrt{3})^2 \cdot \frac{\sqrt{3}}{4} = \frac{3\sqrt{3}}{4}.\]
mathchampion1 2017-02-08 19:22:11
y+z is a 120 degree sector
SomethingNeutral 2017-02-08 19:22:11
x+y = equilateral triangle, y+z = sector (?) of circle
Ani10 2017-02-08 19:22:11
y+z=sector of circle
wiler5002 2017-02-08 19:22:11
y+z is a sector of the cirlce with angle 120
devenware 2017-02-08 19:22:16
We know that $y + z$ is the area of a sector of the circle with angle $120^{\circ}.$ So, $y + z = \dfrac{\pi}{3}.$
abvenkgoo 2017-02-08 19:22:32
$x+y+z = /sqrt{3}$
Arieoreos 2017-02-08 19:22:32
It's half of x+y+z, each triangle is
GeronimoStilton 2017-02-08 19:22:32
$x+y+z$ is $\sqrt{3}$
MSTang 2017-02-08 19:22:32
x+y+z=sqrt(3)
uptownmath 2017-02-08 19:22:33
x+y+z is the sum of the areas of the two triangles
devenware 2017-02-08 19:22:40
We know that $x + y + z$ is twice the area of one of our 30-60-90 triangles, or $x+y+z = \sqrt{3}.$
Ani10 2017-02-08 19:23:07
we have enough to find x
devenware 2017-02-08 19:23:08
So, what is $x$?
SomethingNeutral 2017-02-08 19:23:41
x = sqrt(3)-pi/3
chessapple9 2017-02-08 19:23:41
So $x = \sqrt 3 - (\pi/3)$
CaptainGeo 2017-02-08 19:23:41
sqrt3-pi/3
uptownmath 2017-02-08 19:23:41
$sqrt3-\frac{pi}{3}$
mathchampion1 2017-02-08 19:23:41
sqrt(3)-pi/3
cooljoseph 2017-02-08 19:23:41
$\sqrt3-\frac\pi3$
WW92030 2017-02-08 19:23:41
root 3 - pi/3
bomb427006 2017-02-08 19:23:41
$\sqrt3-\pi/3$
LegoLdr 2017-02-08 19:23:41
root(3)-1/3pi
seanwang2001 2017-02-08 19:23:45
sqrt(3)-pi/3
devenware 2017-02-08 19:23:49
We have $x = (x+y+z) - (y+z) = \sqrt{3} - \dfrac{\pi}{3}.$
devenware 2017-02-08 19:23:50
So what is our answer?
SomethingNeutral 2017-02-08 19:24:23
Answer is E.
Cardinals2014 2017-02-08 19:24:23
E
jybsmartguy2 2017-02-08 19:24:23
E
uptownmath 2017-02-08 19:24:23
$E$
AAANNNMMMIII 2017-02-08 19:24:23
E
FATRaichu 2017-02-08 19:24:23
E
CharlesHong 2017-02-08 19:24:23
E
Knight8 2017-02-08 19:24:23
E
Ani10 2017-02-08 19:24:23
is it E?
gexin 2017-02-08 19:24:23
E
bomb427006 2017-02-08 19:24:23
E
hodori01 2017-02-08 19:24:23
E
devenware 2017-02-08 19:24:27
We have \begin{align*} \frac{x}{x+y} &= \frac{\sqrt{3} - \frac{\pi}{3}}{3\sqrt{3}/4} \\ &= \frac{4\sqrt{3} - \frac{4\pi}{3}}{3\sqrt{3}} \\ &= \frac{4}{3} - \frac{4\pi\sqrt{3}}{27}. \end{align*} Our answer is (E).
cooljoseph 2017-02-08 19:24:46
Yay!
amwmath 2017-02-08 19:24:46
Yay
warrenwangtennis 2017-02-08 19:24:46
easy!
tfz4629 2017-02-08 19:24:46
Yay
mathchampion1 2017-02-08 19:24:46
so easy
devenware 2017-02-08 19:24:53
Or maybe we're just all really good.
LegoLdr 2017-02-08 19:25:09
That's probably it
MSTang 2017-02-08 19:25:09
why not both?
devenware 2017-02-08 19:25:19
We'll try the next one to test our hypotheses.
devenware 2017-02-08 19:25:24
That's the scientific method or something.
devenware 2017-02-08 19:25:42
23. How many triangles with positive area have all their vertices at points $(i,j)$ in the coordinate plane, where $i$ and $j$ are integers between $1$ and $5$, inclusive?
$\phantom{hi}$
$\text{(A) } 2128 \quad
\text{(B) } 2148 \quad
\text{(C) } 2160 \quad
\text{(D) } 2200 \quad
\text{(E) } 2300$
devenware 2017-02-08 19:25:47
How should we approach this problem?
iNomOnCountdown 2017-02-08 19:26:31
Complementary counting
Arieoreos 2017-02-08 19:26:31
Complementary counting
amwmath 2017-02-08 19:26:31
Complementary counting
dipenm 2017-02-08 19:26:31
Complementary counting?
Liopleurodon 2017-02-08 19:26:31
Complementary Counting
RedPhoenix 2017-02-08 19:26:31
complementary
challengetogo 2017-02-08 19:26:31
complementary counting?
thedoge 2017-02-08 19:26:31
use complementary counting
WW92030 2017-02-08 19:26:31
complementary counting
devenware 2017-02-08 19:26:40
We could try complementary counting. That is, let's count all the triples and subtract the triples that give degenerate triangles.
devenware 2017-02-08 19:26:41
Why do we think complementary counting is useful here?
GeneralCobra19 2017-02-08 19:27:37
It would be hard to generate triangles "that work"
Liopleurodon 2017-02-08 19:27:37
Only have to count degenerate triangles
WW92030 2017-02-08 19:27:37
the degenerate triples are collinear points
garretth 2017-02-08 19:27:37
collinear subtraction
AAANNNMMMIII 2017-02-08 19:27:37
degenerate triangles (lines)
WW92030 2017-02-08 19:27:37
There are only a few degenerate triples
Brainiac2 2017-02-08 19:27:37
Not many degenerate answers.
ItzVineeth 2017-02-08 19:27:37
Because we can take what we don't want subtract it from everything and recieve what we wan't
adyj 2017-02-08 19:27:37
It's not as bashy as counting all possible triangles.
DarkPikachu 2017-02-08 19:27:37
there's a lot less cases of degenerate triangles than ones with positive area
brainiac1 2017-02-08 19:27:37
the complementary case only has one case: the points all lie on a line
devenware 2017-02-08 19:27:42
Because degenerate triangles are pretty easy to count.
devenware 2017-02-08 19:27:52
Degenerate triangles have all three vertices are on the same line.
Happy2020 2017-02-08 19:28:30
Draw a diagram ...?
GM2B 2017-02-08 19:28:30
diagram?
mathchampion1 2017-02-08 19:28:30
diagram
ah123 2017-02-08 19:28:30
draw a picture
CaptainGeo 2017-02-08 19:28:30
draw a graph
alcumus1503 2017-02-08 19:28:30
diagram
it1023 2017-02-08 19:28:30
Draw a graph.
devenware 2017-02-08 19:28:32
Okay let's draw the diagram.
devenware 2017-02-08 19:28:36
devenware 2017-02-08 19:28:43
OK, how many triangles are there total in this grid?
rishis 2017-02-08 19:29:44
25 C3=2300
mathchampion1 2017-02-08 19:29:44
25C3 including degenerate
Jayjayliu 2017-02-08 19:29:44
25 choose 3
DarkPikachu 2017-02-08 19:29:44
25 choose 3
Bill9000 2017-02-08 19:29:44
2300
conniejeon1 2017-02-08 19:29:44
2300
tdeng 2017-02-08 19:29:44
$\dbinom{25}{3}$
bobjoe123 2017-02-08 19:29:44
$\binom{25}{3}$
mutinykids 2017-02-08 19:29:44
2300
devenware 2017-02-08 19:29:48
There are 25 vertices and we need to pick 3, so there are \[\dbinom{25}3=\frac{25\cdot24\cdot23}{3\cdot2\cdot1}=25\cdot4\cdot23=2300.\]
mathchampion1 2017-02-08 19:30:30
we can eliminate choice (E)
devenware 2017-02-08 19:30:36
Yeah, so (E) is definitely wrong.
devenware 2017-02-08 19:30:40
Now, how can we organize our counting for all the degenerate triangles?
amwmath 2017-02-08 19:31:17
Cases? By slope, perhaps?
3_14152 2017-02-08 19:31:17
by slope of line
amwmath 2017-02-08 19:31:17
Slope.
PenguinJoe 2017-02-08 19:31:17
different line slopes?
bomb427006 2017-02-08 19:31:17
slope of the line connecting the 3 points
CornSaltButter 2017-02-08 19:31:17
Line slopes
MathMan1234 2017-02-08 19:31:17
slope of the line formed by the points?
Brainiac2 2017-02-08 19:31:17
slopes?
Ani10 2017-02-08 19:31:17
slope of lines
devenware 2017-02-08 19:31:23
Let's count by slope. What slope should we start with?
SomethingNeutral 2017-02-08 19:31:51
0
Talker32332 2017-02-08 19:31:51
0?
jeffshen 2017-02-08 19:31:51
0
challengetogo 2017-02-08 19:31:51
0
Brisingrfire 2017-02-08 19:31:51
0
CrystalEye 2017-02-08 19:31:51
0
FATRaichu 2017-02-08 19:31:51
0
spotty2012 2017-02-08 19:31:51
0
geomstar_1234 2017-02-08 19:31:51
0
orchdork123 2017-02-08 19:31:51
0?
chris100 2017-02-08 19:31:51
0
devenware 2017-02-08 19:31:54
Slope 0:
devenware 2017-02-08 19:31:56
devenware 2017-02-08 19:31:59
I'll get you started. The number of degenerate triangles here is 5 times, um, something. 5 times what?
zjjc123 2017-02-08 19:32:46
5 choose 3
SomethingNeutral 2017-02-08 19:32:46
5c3
dzhou100 2017-02-08 19:32:46
5 choose 3
RedHawk 2017-02-08 19:32:46
5 C 3
pie314159265 2017-02-08 19:32:46
5C3
wiler5002 2017-02-08 19:32:46
$\binom{5}{3}$
chessapple9 2017-02-08 19:32:46
5 choose 3, which is 10.
Smoothfang 2017-02-08 19:32:46
5C3=10
vbcnxm 2017-02-08 19:32:46
5 choose 3
skmc 2017-02-08 19:32:46
5 choose 3
Quaratinium 2017-02-08 19:32:46
5 choose 3 then that times 5
devenware 2017-02-08 19:32:49
To get a degenerate triangle on one of those lines, we pick 3 of the five points.
devenware 2017-02-08 19:32:51
There are $5\dbinom53=5\cdot10=50$ horizontal degenerate triangles.
devenware 2017-02-08 19:32:53
What should we do next?
zjjc123 2017-02-08 19:33:26
slope undefined
RedHawk 2017-02-08 19:33:26
Undefined slope
jeffshen 2017-02-08 19:33:26
undefined
GAN 2017-02-08 19:33:26
vertical
adyj 2017-02-08 19:33:26
slope is undefined
geomstar_1234 2017-02-08 19:33:26
undefined slope?
ShineBunny 2017-02-08 19:33:26
vertical slope
uptownmath 2017-02-08 19:33:26
slope undefined
ilovemath04 2017-02-08 19:33:26
undefined slopes
devenware 2017-02-08 19:33:32
The number of vertical degenerate triangles should be the same, 50.
devenware 2017-02-08 19:33:34
devenware 2017-02-08 19:33:35
What other slopes are possible?
alcumus1503 2017-02-08 19:34:33
1
kiwitrader123 2017-02-08 19:34:33
1
NewbieGamer 2017-02-08 19:34:33
2
dzhou100 2017-02-08 19:34:33
1 and -1
awesome1028717 2017-02-08 19:34:33
slope 1, 1/2, -1, -1/2
iks92 2017-02-08 19:34:33
1/2, 2, 1
jybsmartguy2 2017-02-08 19:34:33
+ or - 1
amwmath 2017-02-08 19:34:33
$\pm2^{\pm1}$ and $\pm1$
hodori01 2017-02-08 19:34:33
1,-1,1/2,-1/2,2,-2
warrenwangtennis 2017-02-08 19:34:33
1/2, -1/2
Picroft 2017-02-08 19:34:33
2, 0.5, -2, -0.5
awesome1028717 2017-02-08 19:34:33
slope 1
Benjy450 2017-02-08 19:34:33
1 and -1
xjeanniewx 2017-02-08 19:34:33
slope 1 and -1
Arieoreos 2017-02-08 19:34:33
slope of 1, slope of 1/2, -1/2, 2, -2
awesome1028717 2017-02-08 19:34:33
slope -1/2
devenware 2017-02-08 19:34:37
We can only get slopes of $\pm1$, $\pm2$, and $\pm\frac 12$.
devenware 2017-02-08 19:34:44
Specifically, we can look at any line with at least 3 points on it in this diagram. If the line isn't vertical or horizontal then the $x$-coordinates form part of an arithmetic progression inside $\{1,2,3,4,5\}$ and the same for the $y$-coordinates. Thus the difference between neighboring points is $\pm1$ or $\pm2$. Therefore the numerator and denominator of the slope are both at most 2.
devenware 2017-02-08 19:34:55
How many lines of slope 2?
devenware 2017-02-08 19:35:38
(I'll note that it'll probably help to be drawing along!)
CornSaltButter 2017-02-08 19:35:42
3
GeneralCobra19 2017-02-08 19:35:42
There are 3 lines
kiwitrader123 2017-02-08 19:35:42
3
WW92030 2017-02-08 19:35:42
3
GeronimoStilton 2017-02-08 19:35:42
$3$
n2001 2017-02-08 19:35:42
3
AAANNNMMMIII 2017-02-08 19:35:42
3
Mario2357 2017-02-08 19:35:42
3
mafsalamander 2017-02-08 19:35:42
3
antmath2520 2017-02-08 19:35:42
3
evanhlu 2017-02-08 19:35:42
3
CaptainGeo 2017-02-08 19:35:42
3
mathcount2002 2017-02-08 19:35:42
3
Goliath 2017-02-08 19:35:42
nope, bleh, 3
acegikmoqsuwy2000 2017-02-08 19:35:42
$3$
devenware 2017-02-08 19:35:46
devenware 2017-02-08 19:35:50
Each of these lines contains one degenerate triangle.
devenware 2017-02-08 19:35:53
There are 3 degenerate triangles of slope 2.
devenware 2017-02-08 19:35:56
What else?
Goliath 2017-02-08 19:36:20
another three for -2
jybsmartguy2 2017-02-08 19:36:20
-2
uptownmath 2017-02-08 19:36:20
slope -2
sbundlab1 2017-02-08 19:36:20
slope of -2
Ani10 2017-02-08 19:36:20
3 of slope -2
Arieoreos 2017-02-08 19:36:20
3 lines slope -2
mutinykids 2017-02-08 19:36:20
and 3 of slope -2
WW92030 2017-02-08 19:36:20
3 lines for -2
jeffshen 2017-02-08 19:36:20
3 for -2 slope
dzhou100 2017-02-08 19:36:20
-2 slope, also 3.
ilovemath04 2017-02-08 19:36:20
3 for -2
oriduck.nk 2017-02-08 19:36:20
-2
Bill9000 2017-02-08 19:36:20
slope of -2
devenware 2017-02-08 19:36:24
Symmetrically, there are 3 degenerate triangles of slope $-2$.
devenware 2017-02-08 19:36:28
tooswagforyou 2017-02-08 19:36:50
slope 1/2
Mario2357 2017-02-08 19:36:50
also 3 of -2,1/2, -1/2
LegoLdr 2017-02-08 19:36:50
3 lines with 1/2 slope too
acegikmoqsuwy2000 2017-02-08 19:36:50
slope -2, 1/2, -1/2 should all also yield 3
amwmath 2017-02-08 19:36:50
$3$ of $-2$, $-\frac12$, and $\frac12$ each as well.
warrenwangtennis 2017-02-08 19:36:50
1/2
mikhailgromov 2017-02-08 19:36:50
another 6 for \pm\frac12
LegoLdr 2017-02-08 19:36:50
Same is true for 1/2 and -1/2
devenware 2017-02-08 19:36:59
There are also 3 degenerate triangles of slope $+\dfrac12$ and 3 of slope $-\dfrac12$.
devenware 2017-02-08 19:37:05
How many lines are there of slope 1?
devenware 2017-02-08 19:37:34
(Look at your diagram carefully! Remember we need at least 3 of our dots on the line.)
mathmagician 2017-02-08 19:37:51
5
evanhlu 2017-02-08 19:37:51
5
jeffshen 2017-02-08 19:37:51
5
sbundlab1 2017-02-08 19:37:51
5
ilovemath04 2017-02-08 19:37:51
5
math101010 2017-02-08 19:37:51
5
uptownmath 2017-02-08 19:37:51
$5$
pratima4 2017-02-08 19:37:51
5
LegoLdr 2017-02-08 19:37:51
5?
kenneo1 2017-02-08 19:37:51
5
dipenm 2017-02-08 19:37:51
no 5
pratima4 2017-02-08 19:37:51
5
kevindk 2017-02-08 19:37:51
5
Blocry 2017-02-08 19:37:51
5
devenware 2017-02-08 19:37:57
devenware 2017-02-08 19:37:59
That top and bottom lines each have 1 degenerate triangle, the second and fourth lines have $\dbinom43=4$, and the middle line has $\dbinom53=10$.
devenware 2017-02-08 19:38:14
There are $1+4+10+4+1=20$ degenerate triangles of slope 1.
kiwitrader123 2017-02-08 19:38:43
same for slope -1
spotty2012 2017-02-08 19:38:43
this is also true for a slope of -1
sbundlab1 2017-02-08 19:38:43
-1 also has 20
FATRaichu 2017-02-08 19:38:43
same for -1
ilovemath04 2017-02-08 19:38:43
20 for -1
uptownmath 2017-02-08 19:38:43
same for slope -1
fields123 2017-02-08 19:38:43
20 for slope -1
CornSaltButter 2017-02-08 19:38:43
Same for slope -1 by symmetry
devenware 2017-02-08 19:38:47
Likewise there are 20 degenerate triangles of slope $-1$.
devenware 2017-02-08 19:38:50
What's the answer?
GeronimoStilton 2017-02-08 19:39:38
It's 2148!
Arieoreos 2017-02-08 19:39:38
2148
CrystalEye 2017-02-08 19:39:38
2148
SmartGuy101 2017-02-08 19:39:38
$\boxed B$
Benjy450 2017-02-08 19:39:38
2148, B
FATRaichu 2017-02-08 19:39:38
2148, B
SmartGuy101 2017-02-08 19:39:38
2148
CrystalEye 2017-02-08 19:39:38
B
GeneralCobra19 2017-02-08 19:39:38
2300-252=2148, (B)
SnakeYu 2017-02-08 19:39:38
B
chessapple9 2017-02-08 19:39:38
I think it is B
amc8nov 2017-02-08 19:39:38
2148!!!
michigo 2017-02-08 19:39:38
2148
amc8nov 2017-02-08 19:39:38
B
chris100 2017-02-08 19:39:38
B
orchdork123 2017-02-08 19:39:38
B
goodball 2017-02-08 19:39:38
B
devenware 2017-02-08 19:39:41
Complementary counting tells us the total number of triangles is \[2300-50-50-3-3-3-3-20-20=\boxed{2148}.\]
devenware 2017-02-08 19:39:42
The answer is (B).
rlzhang 2017-02-08 19:40:06
yay
SmartGuy101 2017-02-08 19:40:06
YAY
devenware 2017-02-08 19:40:14
Well, we've scientifically proven it:
adyj 2017-02-08 19:40:18
So, the conclusion for easy or smartness is...
AAANNNMMMIII 2017-02-08 19:40:18
so easy, we must be really good
pie314159265 2017-02-08 19:40:35
what about experimental error
devenware 2017-02-08 19:40:38
Good point...
devenware 2017-02-08 19:40:41
We better try a few more.
antmath2520 2017-02-08 19:41:30
Can't 'prove' anything scientifically, can only disprove
devenware 2017-02-08 19:41:36
Too philosophical. Next problem.
devenware 2017-02-08 19:41:42
24. For certain real numbers $a$, $b$, and $c$, the polynomial \[g(x) = x^3 +ax^2 + x+10\]has three distinct roots, and each root of $g(x)$ is also a root of the polynomial\[f(x) = x^4 +x^3 +bx^2 + 100x + c.\]What is $f(1)$?
$\phantom{hi}$
$\text{(A) } {-9009} \quad
\text{(B) } {-8008} \quad
\text{(C) } {-7007} \quad
\text{(D) } {-6006} \quad
\text{(E) } {-5005}$
devenware 2017-02-08 19:42:02
This problem is also Problem 23 from the 12A this year. We'll only solve the problem once.
devenware 2017-02-08 19:42:15
The problem tells us that $g(x)$ has three distinct roots, so let's factor it as $g(x) = (x+q)(x+r)(x+s).$
devenware 2017-02-08 19:42:17
How can we factor $f(x)$?
uptownmath 2017-02-08 19:43:06
(x+q)(x+r)(x+s)(x+t)
warrenwangtennis 2017-02-08 19:43:06
g(x)(x-t)
Darth37 2017-02-08 19:43:06
(x + q)(x + r)(x + s)(x + u)
challengetogo 2017-02-08 19:43:06
(x+q)(x+r)(x+s)(x+a)
Wave-Particle 2017-02-08 19:43:06
$f(x) = (x+a)(x+q)(x+r)(x+s)$
Arieoreos 2017-02-08 19:43:06
(x+q)(x+r)(x+s)(x+t)
tdeng 2017-02-08 19:43:06
(x+q)(x+r)(x+s)(x+t) for some t
raxu 2017-02-08 19:43:06
$(x+q)(x+r)(x+s)(x+t)$
CaptainGeo 2017-02-08 19:43:06
(x+q)(x+r)(x+s)(x+m)
SomethingNeutral 2017-02-08 19:43:06
or (x+q)(x+r)(x+s)(x+t)
brainiac1 2017-02-08 19:43:06
$(x+q)(x+r)(x+s)(x+t)$
First 2017-02-08 19:43:06
(x+n)(x+q)(x+r)(x+s)
devenware 2017-02-08 19:43:10
Since each of the roots of $g(x)$ is a root of $f(x)$, we have \[f(x) = (x+q)(x+r)(x+s)(x+t) = g(x)(x+t),\] for some real number $t.$
devenware 2017-02-08 19:43:11
That is,
devenware 2017-02-08 19:43:13
\[x^4+x^3+bx^2+100x+c=(x^3+ax^2+x+10)(x+t).\]
devenware 2017-02-08 19:43:17
Now what?
warrenwangtennis 2017-02-08 19:43:51
expand and equate coefficients
Darth37 2017-02-08 19:43:51
Expand and equate like terms
skmc 2017-02-08 19:43:51
actually multiply through and match the coeficents?
MathMan1234 2017-02-08 19:43:51
just write equations for the coefficients
devenware 2017-02-08 19:43:56
Now we equate coefficients and proceed to rock.
devenware 2017-02-08 19:44:04
Now we equate coefficients and proceed to rock.
devenware 2017-02-08 19:44:06
What does the constant term tell us?
AlisonH 2017-02-08 19:44:43
c=10t
Bill9000 2017-02-08 19:44:43
$c=10t$
amwmath 2017-02-08 19:44:43
$c=10t$
mathchampion1 2017-02-08 19:44:43
10t=c
summitwei 2017-02-08 19:44:43
10t = c
jybsmartguy2 2017-02-08 19:44:43
c=10t
CaptainGeo 2017-02-08 19:44:43
10t=c
PenguinJoe 2017-02-08 19:44:43
awesome1028717 2017-02-08 19:44:43
c=10t
devenware 2017-02-08 19:44:47
The constant term on the left is $c$ and the constant term on the right is $10t$. Thus \[c=10t.\]
devenware 2017-02-08 19:44:59
What does the linear term tell us?
Goliath 2017-02-08 19:45:38
t=90
Darth37 2017-02-08 19:45:38
t = 90
jybsmartguy2 2017-02-08 19:45:38
100=t+10
summitwei 2017-02-08 19:45:38
10+t=100
FATRaichu 2017-02-08 19:45:38
10+t=100
tdeng 2017-02-08 19:45:38
100 = 10+t
abvenkgoo 2017-02-08 19:45:38
$10+t = 100$
spotty2012 2017-02-08 19:45:38
t=90
wiler5002 2017-02-08 19:45:38
$10+t=100$
uptownmath 2017-02-08 19:45:38
10+t=100 or t=90
mohanxue612 2017-02-08 19:45:38
t=90
ilovemath04 2017-02-08 19:45:38
100 = t+10
abvenkgoo 2017-02-08 19:45:38
$t = 90$
brainiac1 2017-02-08 19:45:38
$10+t=100$
Goliath 2017-02-08 19:45:38
10+t=100, t=90
math101010 2017-02-08 19:45:38
t=90
ScienceSpirit 2017-02-08 19:45:38
100=10+t so t=90
devenware 2017-02-08 19:45:43
The linear term on the left is $100$ and the linear term on the right is $10+t$, so \[100=10+t.\]
devenware 2017-02-08 19:46:00
Quadratic?
mathfun5 2017-02-08 19:46:40
at + 1 = b
Darth37 2017-02-08 19:46:40
at + 1 = b
tdeng 2017-02-08 19:46:40
b = at+1
antmath2520 2017-02-08 19:46:40
$b = at + 1$
PenguinJoe 2017-02-08 19:46:40
at + 1 = b
FATRaichu 2017-02-08 19:46:40
at+1=b
brainiac1 2017-02-08 19:46:40
$b=at+1$
AlisonH 2017-02-08 19:46:40
at+1=b
devenware 2017-02-08 19:46:43
Equating the quadratic terms gives us \[b=at+1.\]
devenware 2017-02-08 19:46:44
Cubic?
wiler5002 2017-02-08 19:47:23
$a+t=1$
CrystalEye 2017-02-08 19:47:23
1=a+90
uptownmath 2017-02-08 19:47:23
1=t+a or 1=90+a or a=-89
SomethingNeutral 2017-02-08 19:47:23
t + a = 1
Smoothfang 2017-02-08 19:47:23
a+t=1
amwmath 2017-02-08 19:47:23
$1=t+a$
quanhui868 2017-02-08 19:47:23
a+t=1
devenware 2017-02-08 19:47:28
Equating the cubic terms gives us \[1=a+t.\]
devenware 2017-02-08 19:47:41
So we get the system
devenware 2017-02-08 19:47:42
\begin{align*}
c&=10t\\
100&=10+t\\
b&=at+1\\
1&=a+t\\
\end{align*}
oriduck.nk 2017-02-08 19:47:55
t=90
mathchampion1 2017-02-08 19:47:55
t=90 and find a
Bill9000 2017-02-08 19:48:04
We already know $t=90$
spotty2012 2017-02-08 19:48:04
but we know t=90
devenware 2017-02-08 19:48:07
The second equation tells us that $t=90$.
Goliath 2017-02-08 19:48:17
a=-89
CaptainGeo 2017-02-08 19:48:17
a=-89
PenguinJoe 2017-02-08 19:48:17
a = -89
LegoLdr 2017-02-08 19:48:17
a=-89
Smoothfang 2017-02-08 19:48:17
a=-89
devenware 2017-02-08 19:48:21
The fourth equation tells us that $1=a+90$. Therefore $a=-89$.
devenware 2017-02-08 19:49:02
We could figure out $b$ and $c$, too, but do we need to?
mathforfun2016 2017-02-08 19:49:40
no
uptownmath 2017-02-08 19:49:40
No.
EpicRuler101 2017-02-08 19:49:40
no
SomethingNeutral 2017-02-08 19:49:40
no
GeronimoStilton 2017-02-08 19:49:40
No.
acegikmoqsuwy2000 2017-02-08 19:49:40
nope, just use the factored form
antmath2520 2017-02-08 19:49:40
No, use f(x) = g(x) * (x + t)!
amwmath 2017-02-08 19:49:40
$f(1)=g(1)(t+1)$\
mathfun5 2017-02-08 19:49:40
We can do instead f(1) = g(1)*(1+t)
Benjy450 2017-02-08 19:49:40
no just solve for g(1) then multiply by 91
devenware 2017-02-08 19:49:45
We want to find $f(1)$, which we also know is $g(1)(1+t)$. What does that equal?
SomethingNeutral 2017-02-08 19:50:27
f(1) = (1-89+1+10)(1+90) = (-77)(91) = -7007 C
mathfun5 2017-02-08 19:50:27
-7007
quanhui868 2017-02-08 19:50:27
-7007,C
Darth37 2017-02-08 19:50:27
-7007
quartzgirl 2017-02-08 19:50:27
-7007
uptownmath 2017-02-08 19:50:27
-7007
CrystalEye 2017-02-08 19:50:27
-7007
acegikmoqsuwy2000 2017-02-08 19:50:27
$91(-77)=-7007$
XxkalleknightxXlelxd 2017-02-08 19:50:27
-7007
devenware 2017-02-08 19:50:31
\begin{align*}

f(1)&=g(1)(1+t)\\&=(1+a+1+10)(1+t)\\&=(12-89)(91)=-77\cdot91\\&=-7\cdot11\cdot91=-7\cdot1001\\&=\boxed{-7007}.

\end{align*}
devenware 2017-02-08 19:50:32
The answer is (C).
devenware 2017-02-08 19:50:55
(But I will admit, when I solved this problem, I didn't see that -- I found $b$ and $c$ and added everything up! )
SomethingNeutral 2017-02-08 19:51:24
same here
Goliath 2017-02-08 19:51:24
lol, same, i added it up
oriduck.nk 2017-02-08 19:51:24
me too
goodball 2017-02-08 19:51:24
WOw!!!
devenware 2017-02-08 19:51:34
Okay.
devenware 2017-02-08 19:51:36
NEXT PROBLEM.
devenware 2017-02-08 19:52:17
25. How many integers between 100 and 999, inclusive, have the property that some permutation of its digits is a multiple of 11 between 100 and 999? For example, both 121 and 211 have this property.
$\phantom{hi}$
$\text{(A) } 226 \quad
\text{(B) } 243 \quad
\text{(C) } 270 \quad
\text{(D) } 469 \quad
\text{(E) } 486$
devenware 2017-02-08 19:52:43
What's the divisibility rule for 11 again?
amc8nov 2017-02-08 19:53:44
alternating digits sum to multiples of11
summitwei 2017-02-08 19:53:44
for three-digit number abc, a-b+c
kevindk 2017-02-08 19:53:44
even digits sum - odd digit sum is multiple of 11
Darth37 2017-02-08 19:53:44
alternate digit sum
FATRaichu 2017-02-08 19:53:44
alternating digits
BIGBUBBLE 2017-02-08 19:53:44
alternate digit sums differ by a multiple of 11
GeneralCobra19 2017-02-08 19:53:44
the alternating digits' sum have a difference that mod 11 is 0
adyj 2017-02-08 19:53:44
Alternate adding and subtracting the digits. If result is divisible by 11, the number is.
tdeng 2017-02-08 19:53:44
Alternating sum is divisible by 11
garretth 2017-02-08 19:53:44
a+c = b mod 11
RedHawk 2017-02-08 19:53:44
alternating sum is congruent mod 11
vbcnxm 2017-02-08 19:53:44
alternating numbers add/subtract div by 11?!
devenware 2017-02-08 19:53:48
Since $10\equiv-1\pmod{11}$, a three-digit number $\overline{abc}$ is a multiple of 11 if \[100a+10b+c\equiv(-1)^2a+(-1)b+c\equiv a-b+c\equiv0\pmod{11}.\]
devenware 2017-02-08 19:53:51
That is, we test for divisibility by 11 by checking whether $a-b+c$ is a multiple of 11.
devenware 2017-02-08 19:53:57
Furthermore, since all these are digits, we can only get $a+c=b$ or $a+c=b+11$.
devenware 2017-02-08 19:54:35
So let's always use $b$ to be the number that's "subtracted", so we're looking for three types of numbers

\begin{align*}

\overline{abc}\\

\overline{bac}\\

\overline{acb}\\

\end{align*}

where $a+c\equiv b\pmod{11}$.
devenware 2017-02-08 19:54:40
These will be our three types of numbers. Let's try listing out some examples and see if we find anything interesting.
devenware 2017-02-08 19:54:42
$$\begin{array}{ |c|c|c|c| }
\hline
\overline{abc} & \overline{bac} & \overline{acb} \\
\hline
737 & 377 & 773 \\
484 & 844 & 448 \\
770 & 770 & 707 \\
\hline
\end{array}$$
devenware 2017-02-08 19:54:45
What's going on in that last row?
proztheboss 2017-02-08 19:55:31
first two numbers are the same
chessapple9 2017-02-08 19:55:31
They're the same.
mathchampion1 2017-02-08 19:55:31
all are 770
ilovemath04 2017-02-08 19:55:31
two are the same
devenware 2017-02-08 19:55:42
If one of the digits of a number is 0, it might fall into two different categories. That's something to be careful about.
devenware 2017-02-08 19:55:45
When exactly can a number be in two of these categories?
EpicRuler101 2017-02-08 19:56:47
it only counts as 1 permutation if there is a 0 in the number
jeffshen 2017-02-08 19:56:47
multiple of 10
antmath2520 2017-02-08 19:56:47
Multiple of 110
Goliath 2017-02-08 19:56:47
when it has 0 as a digit
SomethingNeutral 2017-02-08 19:56:47
when it is n-n-0
Brisingrfire 2017-02-08 19:56:47
when it has repeating digits and a zero
PenguinJoe 2017-02-08 19:56:47
multiple of 10 and 11?
devenware 2017-02-08 19:56:54
If $\overline{xyz}$ is of the form $\overline{abc}$ and also of the form $\overline{bac}$ then

\begin{align*}

x-y+z&\equiv0\pmod{11}\\

y-x+z&\equiv0\pmod{11}\\

\end{align*}

which when added tells us that one of the digits is zero. More generally, if a number fits more than one of these patterns then one of the digits has to be zero and the other 2 digits are equal.
devenware 2017-02-08 19:57:08
Also, if all digits are nonzero then a number can only be in one of these forms:

770 has the form $\overline{abc}$ and $\overline{bac}$

707 has the form $\overline{acb}$ and $\overline{bac}$
devenware 2017-02-08 19:57:18
That's something we'll have to worry about along the way.
devenware 2017-02-08 19:57:19
What else is bad about the digit 0 here?
v4913 2017-02-08 19:58:01
It cannot be the first digit.
AlisonH 2017-02-08 19:58:01
it can't be the first digit
uptownmath 2017-02-08 19:58:01
you can't start with it
geomstar_1234 2017-02-08 19:58:01
it can't be 077
adyj 2017-02-08 19:58:01
Cant be in the hundreds place
spotty2012 2017-02-08 19:58:01
it can't be in the first position
bobjoe123 2017-02-08 19:58:01
Cant start with 0
sgadekar 2017-02-08 19:58:01
cant start the number
mathmagician 2017-02-08 19:58:01
077 is not a 3 digit number
mepehe888 2017-02-08 19:58:01
it can't go in the hundredths place
amwmath 2017-02-08 19:58:01
Can't be first digit
skmc 2017-02-08 19:58:01
it can't go as the first digit of the number
mathfun5 2017-02-08 19:58:01
cant be first
MSTang 2017-02-08 19:58:01
cant start a #
mathmagician 2017-02-08 19:58:01
cab doesn't exist
Kirby703 2017-02-08 19:58:01
it can't start off numbers (either the ones we start with or the multiples of 11)
fields123 2017-02-08 19:58:01
it can't be the first digit
ScienceSpirit 2017-02-08 19:58:01
can't have 0bc
challengetogo 2017-02-08 19:58:01
it can't be placed as the hundreds digit
PenguinJoe 2017-02-08 19:58:01
it only has two permutations
Bill9000 2017-02-08 19:58:01
It can't be in the hundreds place!
Brainiac2 2017-02-08 19:58:01
It can't be in front.
evanhlu 2017-02-08 19:58:01
only 2 possibilities
devenware 2017-02-08 19:58:36
0 can't be the first digit since 077 is not a 3-digit number.
devenware 2017-02-08 19:58:51
So let's deal with the digit 0 first. Our strategy will be to find all the numbers that fit our patterns with one of the digits zero first. After that we'll count the numbers that fit our patterns using only nonzero digits.
devenware 2017-02-08 19:59:11
Are there any viable numbers with 2 digits equal to 0?
uptownmath 2017-02-08 19:59:37
no
XxkalleknightxXlelxd 2017-02-08 19:59:37
no
proztheboss 2017-02-08 19:59:37
no
CaptainGeo 2017-02-08 19:59:37
no
mathfun5 2017-02-08 19:59:37
no
goodball 2017-02-08 19:59:37
No.
EpicRuler101 2017-02-08 19:59:37
no
evanhlu 2017-02-08 19:59:37
no
vlwilliam 2017-02-08 19:59:37
no
spotty2012 2017-02-08 19:59:37
no
antmath2520 2017-02-08 19:59:37
No
amc8nov 2017-02-08 19:59:37
NO
edus1 2017-02-08 19:59:37
No
XxkalleknightxXlelxd 2017-02-08 19:59:37
none possible
3_14152 2017-02-08 19:59:37
no
adihaya 2017-02-08 19:59:37
no
devenware 2017-02-08 19:59:54
No. We can't solve $a+c\equiv b\pmod{11}$ when 2 of the digits are 0 (unless all three digits are zero).
devenware 2017-02-08 19:59:56
Now we tackle the $\overline{abc}$ case with $b=0$.
devenware 2017-02-08 20:00:14
How many 3-digit numbers are there of the form $\overline {a0c}$ with $a+c=0$ or $a+c=11$?
brainiac1 2017-02-08 20:00:49
8
uptownmath 2017-02-08 20:00:49
8
Kirby703 2017-02-08 20:00:49
8
PenguinJoe 2017-02-08 20:00:49
8
SomethingNeutral 2017-02-08 20:00:49
8
CrystalEye 2017-02-08 20:00:49
8
3_14152 2017-02-08 20:00:49
8
v4913 2017-02-08 20:00:49
8
mathfun5 2017-02-08 20:00:49
8
LearnAMC 2017-02-08 20:00:49
8
FATRaichu 2017-02-08 20:00:49
8
devenware 2017-02-08 20:00:59
Well $a+c=0$ is impossible, but for every choice of $a$ from 2 to 9, there's exactly one viable digit $c=11-a$. There are 8 of these.
XxkalleknightxXlelxd 2017-02-08 20:01:05
902 803
antmath2520 2017-02-08 20:01:05
8 (209, 308, 407, 506, 605, 704, 803, 902)
mathfun5 2017-02-08 20:01:05
209 308 ... 902
devenware 2017-02-08 20:01:10
There they are listed out.
devenware 2017-02-08 20:01:24
What about the $\overline{acb}$ case with $b=0$?
evanhlu 2017-02-08 20:02:20
8
PenguinJoe 2017-02-08 20:02:20
still 8
JoyceX 2017-02-08 20:02:20
also 8
XxkalleknightxXlelxd 2017-02-08 20:02:20
8
AlisonH 2017-02-08 20:02:20
8
LegoLdr 2017-02-08 20:02:20
same thing
jeffshen 2017-02-08 20:02:20
8 cases
ilovemath04 2017-02-08 20:02:20
same
devenware 2017-02-08 20:02:24
This is the same. We're looking for 3-digit numbers of the form $\overline {ac0}$ with $a+c=0$ or $a+c=11$, which is in a different order but the same sets of numbers.
devenware 2017-02-08 20:02:47
The answer is the same, 8. (290 is an example).
devenware 2017-02-08 20:02:55
Alright, we've handled $b=0$. Now we let $a$ or $c$ be zero.
devenware 2017-02-08 20:03:01
Let's look at the case, $\overline {ab0}$ with $a+0=b$ or $a+0=b+11$. How many numbers have this form?
bao2022 2017-02-08 20:03:35
9.
FATRaichu 2017-02-08 20:03:35
9
amwmath 2017-02-08 20:03:35
9.
First 2017-02-08 20:03:35
9?
XxkalleknightxXlelxd 2017-02-08 20:03:35
9
chessapple9 2017-02-08 20:03:35
There are 9 numbers.
ilovemath04 2017-02-08 20:03:35
9
fields123 2017-02-08 20:03:35
9
Bill9000 2017-02-08 20:03:35
$9$
goodball 2017-02-08 20:03:35
9
devenware 2017-02-08 20:03:40
Now $b+11$ is impossible, but for any $a$ from 1 to 9, we get a viable $b$ (these are the numbers 110, 220, etc.). There are 9 of these.
devenware 2017-02-08 20:03:52
(Those are the ones a lot of you thought I meant earlier. Gotta work on my clarity.)
devenware 2017-02-08 20:04:03
How about $\overline {a0b}$ with $a+0=b$ or $a+0=b+11$?
ItzVineeth 2017-02-08 20:04:37
9 miscount
XxkalleknightxXlelxd 2017-02-08 20:04:37
same, 9
bao2022 2017-02-08 20:04:37
9
LearnAMC 2017-02-08 20:04:37
9
bao2022 2017-02-08 20:04:37
9 still
LegoLdr 2017-02-08 20:04:37
also 9
ilovemath04 2017-02-08 20:04:37
same thing
brainiac1 2017-02-08 20:04:37
9 still
Bill9000 2017-02-08 20:04:37
$9$
SomethingNeutral 2017-02-08 20:04:37
9
Bill9000 2017-02-08 20:04:37
9
goodball 2017-02-08 20:04:37
9 as well.
amwmath 2017-02-08 20:04:37
9 again
evanhlu 2017-02-08 20:04:37
9??
mathfun5 2017-02-08 20:04:37
9
dzhou100 2017-02-08 20:04:37
9
FATRaichu 2017-02-08 20:04:37
same
devenware 2017-02-08 20:04:41
There are also 9 of these (101, 202, etc.).
devenware 2017-02-08 20:04:51
How about $\overline{b0c}$ and $\overline{ba0}$?
ItzVineeth 2017-02-08 20:05:14
9 again.....
mathchampion1 2017-02-08 20:05:14
9
jeffshen 2017-02-08 20:05:14
9
bao2022 2017-02-08 20:05:14
9 each
ilovemath04 2017-02-08 20:05:14
same again
SomethingNeutral 2017-02-08 20:05:14
duplicates
Bill9000 2017-02-08 20:05:14
SAME EXACT THING
evanhlu 2017-02-08 20:05:14
18
Kirby703 2017-02-08 20:05:14
9 for each?
LearnAMC 2017-02-08 20:05:20
18 together
devenware 2017-02-08 20:05:23
In these cases, we have to have $b=c$ or $b=a$. These are the numbers like 101, 202, etc., along with 110, 220, etc. That's 18 more, right?
uptownmath 2017-02-08 20:06:09
No, they are duplicates!
SomethingNeutral 2017-02-08 20:06:09
no already counted them
goodball 2017-02-08 20:06:09
No, they repeat!!
mt492 2017-02-08 20:06:09
no, duplicates
mathfun5 2017-02-08 20:06:09
we already counted those!
goodball 2017-02-08 20:06:09
No, repetitions.
brainiac1 2017-02-08 20:06:14
110 was repeated?
Kirby703 2017-02-08 20:06:14
wait a minute we counted these 18 didn't we :o
devenware 2017-02-08 20:06:17
No! We've already counted these! The $\overline{b0c}$ numbers are also $\overline{a0b}$ numbers and the $\overline{ba0}$ numbers are also the $\overline{ab0}$ numbers. Weird, right?
Bill9000 2017-02-08 20:06:31
No, not really.
devenware 2017-02-08 20:06:35
Not so weird. We noticed that there were going to be some overlaps when one of the digits is zero. We also pointed out that this can't happen in our casework once we move on to nonzero digits.
devenware 2017-02-08 20:06:48
Okay, that's it for the zeroes. How many total numbers have a digit of 0?
mathfun5 2017-02-08 20:07:26
34
conniejeon1 2017-02-08 20:07:26
34
goodball 2017-02-08 20:07:26
34
uptownmath 2017-02-08 20:07:26
34
LegoLdr 2017-02-08 20:07:26
34?
jeffshen 2017-02-08 20:07:26
34
goodball 2017-02-08 20:07:26
34 numbers
CrystalEye 2017-02-08 20:07:26
34
goodball 2017-02-08 20:07:26
There are 34 numbers.
devenware 2017-02-08 20:07:32
There are $8+8+9+9=34$ total numbers with a digit of zero.
devenware 2017-02-08 20:07:46
Now we're restricted to thinking about numbers where none of the digits are 0.
devenware 2017-02-08 20:07:47
How could we construct such a number?
proztheboss 2017-02-08 20:08:31
go one digit at a time
goodball 2017-02-08 20:08:31
Casework?
devenware 2017-02-08 20:08:38
Casework.. Yum.
devenware 2017-02-08 20:08:51
What should we use for cases?
bobjoe123 2017-02-08 20:09:39
Middle Number
mathmagician 2017-02-08 20:09:39
the center number?
wiler5002 2017-02-08 20:09:39
middle digit
Ani10 2017-02-08 20:09:39
the minus plus rule
devenware 2017-02-08 20:09:44
Casework on the form of the number again: We will pick which of the three places to put $b$, the subtracted digit, then choose the three digits.
devenware 2017-02-08 20:09:49
There's a dependence on the digits, though. If you pick 2, the third is mostly determined for you. Which 2 digits should we pick to make things possibly go most smoothly?
devenware 2017-02-08 20:10:13
(Remember, we're always going to call the subtracted number in our 11's trick $b$.)
devenware 2017-02-08 20:10:51
I don't want you to pick actual numbers -- I'm saying which two of $a,b,$ and $c$ should we specify?
uptownmath 2017-02-08 20:11:12
a and c
goodball 2017-02-08 20:11:12
digits a and c
goodball 2017-02-08 20:11:12
Use digits a and c
XxkalleknightxXlelxd 2017-02-08 20:11:12
a and c
v4913 2017-02-08 20:11:12
a and c?
mathislife16 2017-02-08 20:11:12
a and c
conniejeon1 2017-02-08 20:11:12
a and c
FATRaichu 2017-02-08 20:11:12
a,c
proztheboss 2017-02-08 20:11:12
a and c
goodball 2017-02-08 20:11:12
a and c
bao2022 2017-02-08 20:11:12
a and c
devenware 2017-02-08 20:11:17
Let's pick $a$ and $c$. That is, we pick two of the three places of the number and drop nonzero digits in those places. Then we hope that their sum, modulo 11, is a nonzero digit that we can put in the third place.
devenware 2017-02-08 20:11:21
It's most natural to start with $b$ in the middle, so let's count the numbers of the form $\overline{abc}$ with all three digits nonzero and $a+c\equiv b\pmod{11}$.
devenware 2017-02-08 20:11:22
For example, if we let $\overline{abc}=\overline{1b5}$, what are the choices for $b$?
SomethingNeutral 2017-02-08 20:12:10
6
mathmagician 2017-02-08 20:12:10
6
Brisingrfire 2017-02-08 20:12:10
6?
mathfun5 2017-02-08 20:12:10
6
smishra 2017-02-08 20:12:10
6
AlisonH 2017-02-08 20:12:10
6
First 2017-02-08 20:12:10
6?
shubhmit 2017-02-08 20:12:10
6
ilovemath04 2017-02-08 20:12:10
only 6
EpicRuler101 2017-02-08 20:12:10
6
bao2022 2017-02-08 20:12:10
6.
v4913 2017-02-08 20:12:10
It has to be 6
devenware 2017-02-08 20:12:19
The only choice for $b$ is 6.
devenware 2017-02-08 20:12:38
Because we need 1 - b + 5 = 0.
devenware 2017-02-08 20:12:41
Will there ever be two choices for $b$?
GeneralCobra19 2017-02-08 20:13:16
No
ItzVineeth 2017-02-08 20:13:16
no
Kirby703 2017-02-08 20:13:16
no
goodball 2017-02-08 20:13:16
For each pair a and c we pick, b only has one choice.
sahi 2017-02-08 20:13:16
no
LearnAMC 2017-02-08 20:13:16
no
it1023 2017-02-08 20:13:16
NO
fields123 2017-02-08 20:13:16
NO
devenware 2017-02-08 20:13:20
No. There is at most one nonzero digit $b$ that solves $b\equiv a+c\pmod{11}$ no matter what $a$ and $c$ are.
devenware 2017-02-08 20:13:38
Even if $a + c$ is more than 11 -- there's still only one choice.
devenware 2017-02-08 20:13:43
For example, if we had 7b7 what is b?
bobjoe123 2017-02-08 20:14:06
3
bao2022 2017-02-08 20:14:06
3
Bill9000 2017-02-08 20:14:06
$3$
smishra 2017-02-08 20:14:06
3
proztheboss 2017-02-08 20:14:06
3
mathfun5 2017-02-08 20:14:06
3
v4913 2017-02-08 20:14:06
3.
jeffshen 2017-02-08 20:14:06
3
antmath2520 2017-02-08 20:14:06
3
mepehe888 2017-02-08 20:14:06
3
kevindk 2017-02-08 20:14:06
3
evanhlu 2017-02-08 20:14:06
3
geomstar_1234 2017-02-08 20:14:06
3
physangel 2017-02-08 20:14:08
3
amwmath 2017-02-08 20:14:08
$3$
devenware 2017-02-08 20:14:14
Yeah, $b$ would have to be 3 there.
devenware 2017-02-08 20:14:32
So no matter what, we can't have two choices for $b$.
devenware 2017-02-08 20:14:41
Okay what about 5b5, then what is b?
mathfun5 2017-02-08 20:15:19
none
jeffshen 2017-02-08 20:15:19
no
First 2017-02-08 20:15:19
can't happen
uptownmath 2017-02-08 20:15:19
There is no possible choice.
FATRaichu 2017-02-08 20:15:19
none
PenguinJoe 2017-02-08 20:15:19
doesn't work
smishra 2017-02-08 20:15:19
none exist
FATRaichu 2017-02-08 20:15:19
no solution
bobjoe123 2017-02-08 20:15:19
None??
ilovemath04 2017-02-08 20:15:19
no value
antmath2520 2017-02-08 20:15:19
None
Kirby703 2017-02-08 20:15:19
wait -1 and 10 aren't digits
mt492 2017-02-08 20:15:19
not possible
brainiac1 2017-02-08 20:15:19
not possible
physangel 2017-02-08 20:15:19
10, so not possible
AOPSmath68 2017-02-08 20:15:19
not possible
audbear 2017-02-08 20:15:19
None would work (it would be negative one...)
LegoLdr 2017-02-08 20:15:19
it's not possible
devenware 2017-02-08 20:15:30
Yeah, we'd want to use b = 10 or b = -1, but those aren't digits...
devenware 2017-02-08 20:15:40
So sometimes we don't have a possible $b$...
devenware 2017-02-08 20:15:46
When do we not have a possible $b$?
SomethingNeutral 2017-02-08 20:16:19
so if a+c = 10 no possible b.
uptownmath 2017-02-08 20:16:19
When a+c=10
Kirby703 2017-02-08 20:16:19
a+c=10
goodball 2017-02-08 20:16:19
a+c=10
bao2022 2017-02-08 20:16:19
When a + c = 10
jeffshen 2017-02-08 20:16:19
when outer digits add up to 10
mathmagician 2017-02-08 20:16:19
when a+c = 10
evanhlu 2017-02-08 20:16:19
a+c=10
GeneralCobra19 2017-02-08 20:16:19
When a+c=10
CrystalEye 2017-02-08 20:16:19
a+c=10
bobjoe123 2017-02-08 20:16:19
When the sum is 10(of a and c)
Brisingrfire 2017-02-08 20:16:19
when a+c=10
shubhmit 2017-02-08 20:16:19
a+c =10
LegoLdr 2017-02-08 20:16:19
when a+c=10
goodball 2017-02-08 20:16:19
When a+c=10
mathfever 2017-02-08 20:16:19
when the first and last digits sum to 10?
summitwei 2017-02-08 20:16:21
a+c=10
adyj 2017-02-08 20:16:21
When the hundreds and ones add to 10, there is not a possible b.
skywalker321 2017-02-08 20:16:24
when a+c = 10
devenware 2017-02-08 20:16:32
Right, if $a + c = 10$, there is no possible $b$.
devenware 2017-02-08 20:16:35
But is that the only case?
mathfun5 2017-02-08 20:17:07
what about a+c = 11 then b cant b 0 cause we already counted that
Ani10 2017-02-08 20:17:07
if a+c=11
Archos 2017-02-08 20:17:07
11
SnakeYu 2017-02-08 20:17:07
what if it adds to 11?
mathislife16 2017-02-08 20:17:07
No, because we've eliminated 0s
devenware 2017-02-08 20:17:18
Right, we can't have $a + c = 11$ either, because we're dealing with all nonzero digits now.
devenware 2017-02-08 20:17:39
Okay, so we want to count how many pairs there are of $a,c$ that add up to something other than 10 or 11.
devenware 2017-02-08 20:17:45
Suggestions?
SomethingNeutral 2017-02-08 20:18:06
complementary?
Bill9000 2017-02-08 20:18:06
COMPLEMENTARY COUNTING!!!
mathfever 2017-02-08 20:18:06
complementary counting?
mt492 2017-02-08 20:18:06
complementary counting
RedHawk 2017-02-08 20:18:06
Complementary counting?
PenguinJoe 2017-02-08 20:18:06
complementary counting
antmath2520 2017-02-08 20:18:06
Complementary counting?
sgadekar 2017-02-08 20:18:14
complementary counting
devenware 2017-02-08 20:18:15
COMPLEMENTARY COUNTING!
devenware 2017-02-08 20:18:21
Without the sum restriction, how many ordered pairs $a$ and $c$ can we pick?
conniejeon1 2017-02-08 20:18:51
81
SomethingNeutral 2017-02-08 20:18:51
9^2 = 81
mathmagician 2017-02-08 20:18:51
81
FATRaichu 2017-02-08 20:18:51
81
PenguinJoe 2017-02-08 20:18:51
81
adyj 2017-02-08 20:18:51
81
v4913 2017-02-08 20:18:51
81?
jeffshen 2017-02-08 20:18:51
81
LearnAMC 2017-02-08 20:18:51
81
uptownmath 2017-02-08 20:18:51
81 cuz you can't have zeros!
Kirby703 2017-02-08 20:18:51
oh so that's what that's called... 81
devenware 2017-02-08 20:18:54
There are 9 choices for $a$ and 9 for $c$, so 81 total pairs.
devenware 2017-02-08 20:19:02
Okay, how many of those pairs add to 10?
FATRaichu 2017-02-08 20:19:32
9
SomethingNeutral 2017-02-08 20:19:32
9.
Kirby703 2017-02-08 20:19:32
9
jeffshen 2017-02-08 20:19:32
9
WW92030 2017-02-08 20:19:32
9
RedHawk 2017-02-08 20:19:32
9
uptownmath 2017-02-08 20:19:32
9
mathfun5 2017-02-08 20:19:32
9
ilovemath04 2017-02-08 20:19:32
9
v4913 2017-02-08 20:19:32
9
LearnAMC 2017-02-08 20:19:32
9
goodball 2017-02-08 20:19:32
9
mathfever 2017-02-08 20:19:32
9?
physangel 2017-02-08 20:19:32
9
evanhlu 2017-02-08 20:19:32
9
mathfever 2017-02-08 20:19:32
9
LegoLdr 2017-02-08 20:19:32
9
Blocry 2017-02-08 20:19:32
9
devenware 2017-02-08 20:19:39
For any nonzero digit $a$, the value $c=10-a$ is also a nonzero digit, so there are 9 invalid pairs that add to 10.
devenware 2017-02-08 20:19:40
How many pairs add to 11?
fields123 2017-02-08 20:20:24
8
amburger66 2017-02-08 20:20:24
8
harry1234 2017-02-08 20:20:24
8
EpicRuler101 2017-02-08 20:20:24
8
brainiac1 2017-02-08 20:20:24
8
Brisingrfire 2017-02-08 20:20:24
8
sahi 2017-02-08 20:20:24
8
Blocry 2017-02-08 20:20:24
8
First 2017-02-08 20:20:24
8
AlisonH 2017-02-08 20:20:24
8
AnaT129 2017-02-08 20:20:24
8
PenguinJoe 2017-02-08 20:20:24
8
devenware 2017-02-08 20:20:31
For any digit $a>1$, the value $c=11-a$ is also a nonzero digit, so there are 8 invalid pairs that add to 11.
devenware 2017-02-08 20:20:32
So how many valid pairs are there?
Kirby703 2017-02-08 20:21:24
64
v4913 2017-02-08 20:21:24
64
goodball 2017-02-08 20:21:24
64
harry1234 2017-02-08 20:21:24
64
mathfun5 2017-02-08 20:21:24
64
conniejeon1 2017-02-08 20:21:24
64
sgadekar 2017-02-08 20:21:24
64
Brisingrfire 2017-02-08 20:21:24
81-17=64
Blocry 2017-02-08 20:21:24
64
physangel 2017-02-08 20:21:24
64
adyj 2017-02-08 20:21:24
81-17=64
AlisonH 2017-02-08 20:21:24
64
mathmagician 2017-02-08 20:21:24
81-9-8 or 64
Bill9000 2017-02-08 20:21:24
64! (Not 64 factorial, just saying.)
bao2022 2017-02-08 20:21:24
64.
mshanmugam 2017-02-08 20:21:24
64
Dolphin8 2017-02-08 20:21:24
64
devenware 2017-02-08 20:22:07
There are $81-9-8=64$ total valid pairs $a$ and $c$.
devenware 2017-02-08 20:22:44
What about numbers of the form $\overline{acb}$ with nonzero digits? How many of those are there?
mathfun5 2017-02-08 20:23:23
64 same!
mathmagician 2017-02-08 20:23:23
also 64
FATRaichu 2017-02-08 20:23:23
64
goodball 2017-02-08 20:23:23
same.
fields123 2017-02-08 20:23:23
same
conniejeon1 2017-02-08 20:23:23
64
goodball 2017-02-08 20:23:23
64 as well
SomethingNeutral 2017-02-08 20:23:23
64.
mathfun5 2017-02-08 20:23:23
a+c = b so same
Blocry 2017-02-08 20:23:23
same?
mathislife16 2017-02-08 20:23:23
Still 64 right? It doesn't change, because there are no 0s
LearnAMC 2017-02-08 20:23:23
64
lemonpower 2017-02-08 20:23:23
same as abc?
devenware 2017-02-08 20:23:27
It's the same! We just take an $\overline{abc}$ number and swap the last two digits. There are 64 of these.
devenware 2017-02-08 20:23:28
What about $\overline{bac}$ with nonzero digits?
devenware 2017-02-08 20:23:38
What about $\overline{bac}$ with nonzero digits?
mathfun5 2017-02-08 20:24:01
64
mathmagician 2017-02-08 20:24:01
same thing! 64
Kirby703 2017-02-08 20:24:01
another 64 for bac
SomethingNeutral 2017-02-08 20:24:01
64
mathchampion1 2017-02-08 20:24:01
64
mathchampion1 2017-02-08 20:24:01
same as well
ilovemath04 2017-02-08 20:24:01
SAME AGAIN
jeffshen 2017-02-08 20:24:01
64
AnaT129 2017-02-08 20:24:03
still 64?
LegoLdr 2017-02-08 20:24:04
same
devenware 2017-02-08 20:24:11
That's the same again. This time we take an $\overline{abc}$ number and swap the first two digits. There are still 64.
devenware 2017-02-08 20:24:14
How many total numbers do we have with all nonzero digits?
bobjoe123 2017-02-08 20:25:03
192
Goliath 2017-02-08 20:25:03
192
CharlesHong 2017-02-08 20:25:03
192
shubhmit 2017-02-08 20:25:03
192
bao2022 2017-02-08 20:25:03
192
CrystalEye 2017-02-08 20:25:03
192
AOPSmath68 2017-02-08 20:25:03
192?
devenware 2017-02-08 20:25:07
There are 3 places to put $b$ and for each of these 3 choices, there are 64 possible 3-digit numbers. We showed at the beginning that these cases don't overlap. Therefore there are $3\cdot64=192$ 3-digit numbers that are permutations of multiples of 11 with no digit zero.
devenware 2017-02-08 20:25:08
And what is the final answer?
guo_fang 2017-02-08 20:25:41
226
mathfun5 2017-02-08 20:25:41
226
FATRaichu 2017-02-08 20:25:41
226
bao2022 2017-02-08 20:25:41
(A)
LegoLdr 2017-02-08 20:25:41
A!!!
goodball 2017-02-08 20:25:41
A
fields123 2017-02-08 20:25:41
226
jeffshen 2017-02-08 20:25:41
A
goodball 2017-02-08 20:25:41
226
mathfun5 2017-02-08 20:25:41
192+34 = 226
mathfun5 2017-02-08 20:25:41
226!
Brisingrfire 2017-02-08 20:25:41
192+34= 226 (A!)
ilovemath04 2017-02-08 20:25:41
226, A
SnakeYu 2017-02-08 20:25:41
226, A
bao2022 2017-02-08 20:25:41
(A) 226
physangel 2017-02-08 20:25:41
A
CrystalEye 2017-02-08 20:25:41
226
CharlesHong 2017-02-08 20:25:41
226 A
First 2017-02-08 20:25:41
$\boxed{\textbf{A}}$!
devenware 2017-02-08 20:25:44
Adding in the numbers with 0 as a digit we find $34+192=\boxed{226}$ total numbers that are permutations of multiples of 11. The answer is (A).
devenware 2017-02-08 20:26:22
WOOHOO!
mathmagician 2017-02-08 20:26:43
WE DID IT
Bill9000 2017-02-08 20:26:43
YAY!!!!!!!!!!!!!!!!!!!!!!!!!!!!!!!!!!!!!!!!!!!!!!!!!!!!!!!!!!!!!!!!!!!!!!!!!!!!!!!!!!
goodball 2017-02-08 20:26:43
That was GREAT!
RemiaFlower 2017-02-08 20:26:52
AWESOME!!!
v4913 2017-02-08 20:26:52
IT WAS AWESOME!
devenware 2017-02-08 20:26:56
AWESOME!!
EpicRuler101 2017-02-08 20:27:05
YAY THANKS SO MUCH
ilovemath04 2017-02-08 20:27:05
AWESOME
devenware 2017-02-08 20:27:16
Alright, everyone ready for the AMC 12?
AnaT129 2017-02-08 20:27:39
YEAHHHHH
amwmath 2017-02-08 20:27:39
YEAH!
First 2017-02-08 20:27:39
Yes!
Bill9000 2017-02-08 20:27:39
GREAT!!!!
PenguinJoe 2017-02-08 20:27:39
YAS
LegoLdr 2017-02-08 20:27:39
YEAH!!!
mathchampion1 2017-02-08 20:27:39
YES!!!!!!!!!!!!!!!!!!!!!!!!!!!!!!!!!!!!!!!!!!!!!!!!!!!!!!!
brainiac1 2017-02-08 20:27:39
Yep
goodball 2017-02-08 20:27:39
YES!!!!!!!!!!!!!!!!!!!
challengetogo 2017-02-08 20:27:39
YESSSS
amwmath 2017-02-08 20:27:39
Le''s DO this!
devenware 2017-02-08 20:27:44
YEAH, LET'S DO THIS!
devenware 2017-02-08 20:27:56
21. A set $S$ is constructed as follows. To begin, $S=\{0,10\}$. Repeatedly, as long as possible, if $x$ is an integer root of some polynomial $a_nx^n+a_{n-1}x^{n-1}+\cdots + a_1x+a_0$ for some $n\ge 1$, all of whose coefficients $a_i$ are elements of $S$,then $x$ is put into $S$. When no more elements can be added to $S$, how many elements does $S$ have?
$\phantom{hi}$
$
\text{(A) } 4 \quad
\text{(B) } 5 \quad
\text{(C) } 7 \quad
\text{(D) } 9 \quad
\text{(E) } 11
$
devenware 2017-02-08 20:28:01
OK, important AMC moment. What relevant theorem haven't we used yet that we often get to see on the AMCs?
First 2017-02-08 20:29:07
Rational Root Theorem
mathcrazymj 2017-02-08 20:29:07
rational root thm
wiler5002 2017-02-08 20:29:07
Rational root theorem
summitwei 2017-02-08 20:29:07
rational root?
devenware 2017-02-08 20:29:12
The Rational Root Theorem!
devenware 2017-02-08 20:29:13
If we were to build a polynomial here with coefficients 0 and 10 what do we know about its roots?
wiler5002 2017-02-08 20:30:15
must be divisors of 10
summitwei 2017-02-08 20:30:15
can only have roots of +- 1, 2, 5, 10
VinVinB 2017-02-08 20:30:15
Factors of 10
mathfun5 2017-02-08 20:30:15
A_n = 10 and a_0 = 10 or 0
summitwei 2017-02-08 20:30:15
can only have roots of +/- 1, 2, 5, 10, 0
devenware 2017-02-08 20:30:21
Well, if $a_0=0$ then we can divide by $x$ and not change the nozero roots, so we should assume that $a_0=10$.
devenware 2017-02-08 20:30:22
The Rational Root Theorem tells us that any integer root is a (positive or negative) factor of 10.
devenware 2017-02-08 20:30:28
What roots can we get with only coefficients of 10?
devenware 2017-02-08 20:31:06
Someone give me a polynomial and a root we can get from it.
VinVinB 2017-02-08 20:31:56
10x + 10 gives -1
CrystalEye 2017-02-08 20:31:56
10x+10=0, x=-1
Figpaste 2017-02-08 20:31:56
10x+10 , x=-1
wiler5002 2017-02-08 20:31:56
$10x+10$ gives $-1$
summitwei 2017-02-08 20:31:56
10x+10, -1
challengetogo 2017-02-08 20:31:56
10x + 10 => x = -1
devenware 2017-02-08 20:32:00
$10x+10$ has root $-1$. (Actually that's the only new number we can add to $S$ so far.)
devenware 2017-02-08 20:32:21
[For example, we can't use $x + 10$ or something like that -- because we don't have a coefficient 1 yet.]
amwmath 2017-02-08 20:32:38
Wait, what about $-10$?
mathfun5 2017-02-08 20:32:38
Why not 1 or 2,5,10
devenware 2017-02-08 20:33:02
Since we can only use 10 as a coefficient at this point, we can't get -10. To get -10, you'd want something like x + 10.
devenware 2017-02-08 20:33:22
So anyway, we can get $-1$ using $10x + 10$, so let's up our set $S$.
devenware 2017-02-08 20:33:23
Now $S=\{-1,0,10\}$.
devenware 2017-02-08 20:34:49
What else can we add to $S$?
LearnAMC 2017-02-08 20:35:22
1
SomethingNeutral 2017-02-08 20:35:22
1?
devenware 2017-02-08 20:35:27
Can you find a polynomial with root 1?
devenware 2017-02-08 20:36:00
Remember, we can only use -1 and 10 as our coefficients at this point.
mathfun5 2017-02-08 20:36:14
what about 0
devenware 2017-02-08 20:36:18
And 0.
Damalone 2017-02-08 20:36:39
10x^10-x^9-x^8...-x-1 gives x=1
FATRaichu 2017-02-08 20:36:39
-x^10-x^9-x^8.....x+10=0
summitwei 2017-02-08 20:36:39
-1x^10-1x^9-1x^8...-1x^2-1x+10, 1
devenware 2017-02-08 20:36:42
The number 1 is a root of \begin{align*}

&\phantom{+}\ \ (-1)x^{10}+(-1)x^{9}+(-1)x^{8}+(-1)x^{7}+(-1)x^{6}\\&+(-1)x^{5}+(-1)x^{4}+(-1)x^{3}+(-1)x^{2}+(-1)x^{1}+10.\end{align*}
devenware 2017-02-08 20:36:44
More intuitively, to have a root 1, a polynomial needs its coefficients to sum to 0. If our coefficients come from $\{-1, 0, 10\}$, we can just choose ten $-1$s and one $10$.
devenware 2017-02-08 20:36:46
Now we have $S=\{-1,0,1,10\}$.
devenware 2017-02-08 20:37:03
I bet we can get $-10$. It's hard for me to imagine that $S$ won't be symmetric (containing only pairs $\pm n$) after all... Can you construct a polynomial that gives us $-10$?
amwmath 2017-02-08 20:37:50
$-10$ from $x+10$
mathman3880 2017-02-08 20:37:50
x+10
summitwei 2017-02-08 20:37:50
1x+10
ToneLoke 2017-02-08 20:37:50
X+10
abvenkgoo 2017-02-08 20:37:50
$x+10$
acegikmoqsuwy2000 2017-02-08 20:37:50
$x+10$
xiongxiong 2017-02-08 20:37:50
X+10
FATRaichu 2017-02-08 20:37:50
x+10=0
CrystalEye 2017-02-08 20:37:50
x+10=0
wiler5002 2017-02-08 20:37:50
$x+10$
CharlesHong 2017-02-08 20:37:50
x+10
amburger66 2017-02-08 20:37:50
x+10 lol
mathfun5 2017-02-08 20:37:50
x+10 = 0
Figpaste 2017-02-08 20:37:50
x+10
LegoLdr 2017-02-08 20:37:52
just x + 10
devenware 2017-02-08 20:37:54
$x+10$ has root $-10$.
devenware 2017-02-08 20:38:17
Indeed, since 1 is in $S$, if $s$ is any element of $S$ we can write the polynomial\[x+s\] with root $-s$.
devenware 2017-02-08 20:38:25
Alright, so now we should see if we can include $\pm2$ and $\pm5$.
devenware 2017-02-08 20:38:26
Is there a nice polynomial with 2 as a root?
summitwei 2017-02-08 20:39:41
x^3+x-10
amwmath 2017-02-08 20:39:41
$x^3+x-10$.
mathman3880 2017-02-08 20:39:41
x^3 + x - 10
Bill9000 2017-02-08 20:39:41
x^3+x-10=0
wiler5002 2017-02-08 20:39:41
$-x^3-x+10$
Bill9000 2017-02-08 20:39:41
$x^3-x-10$
vbcnxm 2017-02-08 20:39:41
-x^3-x+10
wiler5002 2017-02-08 20:39:41
or $x^3+x-10$
challengetogo 2017-02-08 20:39:41
x^3 + x - 10
raxu 2017-02-08 20:39:41
$x^3+x-10=0$
mathcount2002 2017-02-08 20:39:41
x^3+0x^2+x-10=0
FATRaichu 2017-02-08 20:39:41
x^3+x-10=0
devenware 2017-02-08 20:39:44
The binary expansion for 10 is $10=2+2^3$, so 2 must be a root of \[x^3+x-10.\]
devenware 2017-02-08 20:39:46
Therefore $S$ contains $\pm2$.
devenware 2017-02-08 20:39:47
What about 5? Do you see a way to get 5?
summitwei 2017-02-08 20:40:32
2x-10
FATRaichu 2017-02-08 20:40:32
2x-10=0
amwmath 2017-02-08 20:40:32
$2x-10$ now.
acegikmoqsuwy2000 2017-02-08 20:40:32
$2x-10$
wiler5002 2017-02-08 20:40:32
$2x-10$
drazn98 2017-02-08 20:40:32
2x-10
VinVinB 2017-02-08 20:40:32
2x-10
xiongxiong 2017-02-08 20:40:32
2x-10
SomethingNeutral 2017-02-08 20:40:32
2x-10=0?
math-rules 2017-02-08 20:40:32
2x-10
mathman3880 2017-02-08 20:40:32
2x-10
Bill9000 2017-02-08 20:40:32
2x-10=0
AidanNReilly 2017-02-08 20:40:32
2x-10
Figpaste 2017-02-08 20:40:32
2x-10
mathislife16 2017-02-08 20:40:32
2x-10
arandomperson123 2017-02-08 20:40:32
2x-10 => x=5 works
Benjy450 2017-02-08 20:40:32
2x-10
devenware 2017-02-08 20:40:36
Now that we have 2, getting 5 is easy:\[2x-10\] has 5 as a root.
devenware 2017-02-08 20:40:39
We're at the point where $S=\{0,\pm1,\pm2,\pm5,\pm10\}$. Now what elements can we add?
wiler5002 2017-02-08 20:41:13
none
summitwei 2017-02-08 20:41:13
no more by rrt
LearnAMC 2017-02-08 20:41:13
none
Ani10 2017-02-08 20:41:13
nothing
mathfun5 2017-02-08 20:41:13
None cause of rational root theorem
FATRaichu 2017-02-08 20:41:13
none
AnaT129 2017-02-08 20:41:13
none more because that is all the factors of 10
First 2017-02-08 20:41:13
I think none
wiler5002 2017-02-08 20:41:13
because any new memebers must divide a current numnber and all the primes are in there
Figpaste 2017-02-08 20:41:13
none
xiongxiong 2017-02-08 20:41:13
None
mathman3880 2017-02-08 20:41:13
Nothing
arandomperson123 2017-02-08 20:41:16
by the RRT, we can not add any more...
RemiaFlower 2017-02-08 20:41:20
none
devenware 2017-02-08 20:41:23
We can't add anything! Since $a_0$ has to be an element of $S$--and we can assume it is nonzero--any new element we add to $S$ has to be a factor of one of these numbers (and thus a factor of 10). We've already found all the factors of 10 so we're done!
devenware 2017-02-08 20:41:27
The answer?
WW92030 2017-02-08 20:41:48
D
CrystalEye 2017-02-08 20:41:48
9
FATRaichu 2017-02-08 20:41:48
9
LearnAMC 2017-02-08 20:41:48
D
Figpaste 2017-02-08 20:41:48
D
goodball 2017-02-08 20:41:48
D
Benjy450 2017-02-08 20:41:48
9, D
Knight8 2017-02-08 20:41:48
D
brainiac1 2017-02-08 20:41:48
9
goodball 2017-02-08 20:41:48
9
ToneLoke 2017-02-08 20:41:48
D
arandomperson123 2017-02-08 20:41:48
9!!!
devenware 2017-02-08 20:41:55
The answer is 9, (D).
devenware 2017-02-08 20:42:02
devenware 2017-02-08 20:42:25
Okay, next problem.
RemiaFlower 2017-02-08 20:42:33
YAY!!!!!
goodball 2017-02-08 20:42:33
YAY!
RemiaFlower 2017-02-08 20:42:33
YAY!!!!!!
devenware 2017-02-08 20:42:39
The next one was my favorite too.
mathchampion1 2017-02-08 20:42:54
I love states!
devenware 2017-02-08 20:43:02
California is my favorite.
mathchampion1 2017-02-08 20:43:08
Particles!
devenware 2017-02-08 20:43:23
The Charm quark is my favorite.
AnaT129 2017-02-08 20:43:40
ohhh that is a cool one
devenware 2017-02-08 20:43:42
22. A square is drawn in the Cartesian coordinate plane with vertices at $(2,2)$, $(-2,2)$, $(-2,-2)$, and $(2,-2)$. A particle starts at $(0,0)$. Every second it moves with equal probability to one of the eight lattice points (points with integer coordinates) closest to its current position, independently of its previous moves. In other words, the probability is $\dfrac18$ that the particle will move from $(x,y)$ to each of $(x,y+1)$, $(x+1,y+1)$, $(x+1,y)$, $(x+1,y-1)$, $(x,y-1)$, $(x-1,y-1)$, $(x-1,y)$, or $(x-1,y+1)$. The particle will eventually hit the square for the first time, either at one of the 4 corners of the square or at one of the 12 lattice points in the interior of one of the sides of the square. The probability that it will hit at a corner rather than at an interior point of a side is $\dfrac{m}{n}$, where $m$ and $n$ are relatively prime integers. What is $m+n$?
$\phantom{hi}$
$\text{(A) } 4 \quad
\text{(B) } 5 \quad
\text{(C) } 7 \quad
\text{(D) } 15 \quad
\text{(E) } 39$
devenware 2017-02-08 20:43:47
Alright, what's the perfect tool for this problem?
Ani10 2017-02-08 20:44:20
graph paper
quartzgirl 2017-02-08 20:44:20
graph paper
mathfun5 2017-02-08 20:44:20
graph paper and a picture!
Eyed 2017-02-08 20:44:29
graph paper
devenware 2017-02-08 20:44:31
You're right graph paper is super useful here!
devenware 2017-02-08 20:44:36
I hope you brought some to the test!
devenware 2017-02-08 20:44:52
Okay, now what's a good mathematical tool we can use?
wiler5002 2017-02-08 20:45:23
States
mathchampion1 2017-02-08 20:45:23
State diagrams!!!!
BobaFett101 2017-02-08 20:45:23
events with states
math-rules 2017-02-08 20:45:23
state diagram
devenware 2017-02-08 20:45:33
This is a problem where we want to think about states. We want to think about all of the possible places that the particle could be at a given time. Let's draw the diagram:
devenware 2017-02-08 20:45:34
devenware 2017-02-08 20:45:39
So there are 25 different states: the particle could be at any one of these 25 locations.
devenware 2017-02-08 20:45:41
Gosh, 25 is a lot, huh?
xiongxiong 2017-02-08 20:46:11
Symmetry
abvenkgoo 2017-02-08 20:46:11
symmetry though!
BobaFett101 2017-02-08 20:46:11
symmetry
mathislife16 2017-02-08 20:46:11
you can reduce it because of symmetry
mathchampion1 2017-02-08 20:46:11
but it is symmetric around the 12 and 13th point!
drazn98 2017-02-08 20:46:11
symmetry
math-rules 2017-02-08 20:46:11
symmetry
devenware 2017-02-08 20:46:17
A lot of the states are basically the same. The particle wins when it gets to an outer corner, it loses on the outer edge, the inner corners are all fundamentally the same and so are the inner edges (we'll define "same" in a moment).
devenware 2017-02-08 20:46:18
devenware 2017-02-08 20:46:23
Now in a state diagram, we usually label the states with some values. What values should we use to label these states?
First 2017-02-08 20:46:48
probabilities
math-rules 2017-02-08 20:46:48
probability of winning from that point
devenware 2017-02-08 20:46:59
Cool. Any probabilities we know for sure already?
xiongxiong 2017-02-08 20:47:36
0 red, 1 green
tdeng 2017-02-08 20:47:36
Green = 1
wiler5002 2017-02-08 20:47:36
corners are 1
brainiac1 2017-02-08 20:47:36
the probability in the corner is 1
wiler5002 2017-02-08 20:47:36
edges are 0
tdeng 2017-02-08 20:47:36
Red = 0, Green =1
ToneLoke 2017-02-08 20:47:36
red = 0, green = 1
garretth 2017-02-08 20:47:36
corners and edges
xiongxiong 2017-02-08 20:47:36
Green is 1, red is 0
devenware 2017-02-08 20:47:46
A particle at an outer corner has already won! Those get a probability of 1. Likewise a particle at an outer edge has already lost so we put 0 there.
devenware 2017-02-08 20:47:48
devenware 2017-02-08 20:47:52
Oh, and by the way, we can give a good definition for "the same" now. We colored these dots the same color when we know, by symmetry, that they have the same probability of success.
devenware 2017-02-08 20:47:57
It looks like we have three more colors. I don't see immediately what the probabilities at those points are, so let's give them variables:
devenware 2017-02-08 20:47:59
devenware 2017-02-08 20:48:05
I picked $o$ for origin, $c$ for corner, and $e$ for edge, since good variable names are good.
devenware 2017-02-08 20:48:11
Do we have an expression for $o$?
devenware 2017-02-08 20:48:40
Think: What type of state can you move to from $o$?
BobaFett101 2017-02-08 20:48:59
e or c
garretth 2017-02-08 20:48:59
e or c
RemiaFlower 2017-02-08 20:49:08
e or c
devenware 2017-02-08 20:49:09
We can move to $e$ or $c$.
devenware 2017-02-08 20:49:16
Can you use that to write an expression for $o$?
tdeng 2017-02-08 20:49:31
o = 1/2e+1/2c
ToneLoke 2017-02-08 20:49:31
c/2 + e/2
math-rules 2017-02-08 20:49:31
o = (c+e)/2
stronto 2017-02-08 20:49:31
e/2 + c/2
brainiac1 2017-02-08 20:49:31
$o=\frac{1}{2} c+\frac{1}{2} e$
wiler5002 2017-02-08 20:49:31
$\frac{e}{2}+\frac{c}{2}$
amwmath 2017-02-08 20:49:31
$\frac{c+e}2$
Blocry 2017-02-08 20:49:31
0.5e+0.5c
amwmath 2017-02-08 20:49:31
$\frac{4e+4c}8=\frac{e+c}2$
summitwei 2017-02-08 20:49:31
o=1/2*e+1/2*c
goodball 2017-02-08 20:49:31
so the expression is (c+e)/2
devenware 2017-02-08 20:49:35
A particle standing at the middle vertex will go to a $c$ vertex half the time and an $e$ vertex the other half the time. Therefore \[o=\frac12c+\frac12e.\]
devenware 2017-02-08 20:49:38
I like to clear denominators, so \[2o=c+e.\]
devenware 2017-02-08 20:49:42
Can we get an expression for $e$?
devenware 2017-02-08 20:50:31
Let's think about this. What types of states can $e$ move to?
LearnAMC 2017-02-08 20:51:05
0, c, or o
mathmagician 2017-02-08 20:51:05
lose, c, and o
math-rules 2017-02-08 20:51:11
c, o, e, or 0
FATRaichu 2017-02-08 20:51:11
o, c, 0
devenware 2017-02-08 20:51:26
Great. What's the probability that you move to a 0?
CrystalEye 2017-02-08 20:51:41
3/8
WW92030 2017-02-08 20:51:41
3/8
dr3463 2017-02-08 20:51:41
3/8
mathcount2002 2017-02-08 20:51:41
3/8
mathfun5 2017-02-08 20:51:41
3/8
devenware 2017-02-08 20:51:49
Cool. Probability you move to another $e$?
CrystalEye 2017-02-08 20:52:03
1/4
FATRaichu 2017-02-08 20:52:03
1/4
mathcount2002 2017-02-08 20:52:03
1/4
goodball 2017-02-08 20:52:03
1/4
Eyed 2017-02-08 20:52:03
1/4
Bill9000 2017-02-08 20:52:03
1/4
adyj 2017-02-08 20:52:03
1/4
devenware 2017-02-08 20:52:14
Awesome. Probability you move to a $c$?
CrystalEye 2017-02-08 20:52:33
1/4.
mathfun5 2017-02-08 20:52:33
1/4
wiler5002 2017-02-08 20:52:33
also 1/4
dr3463 2017-02-08 20:52:33
2/8
mathfever 2017-02-08 20:52:33
1/4
CharlesHong 2017-02-08 20:52:33
1/4
PenguinJoe 2017-02-08 20:52:33
1/4
BooBooTM 2017-02-08 20:52:33
1/4
mathchampion1 2017-02-08 20:52:34
1/4 again
devenware 2017-02-08 20:52:45
And there's a 1/8 probability that we move to the $o$.
devenware 2017-02-08 20:52:51
So can you write an expression for $e$?
Figpaste 2017-02-08 20:53:21
e = 1/4 e + 1/8 o +1/4 c
garretth 2017-02-08 20:53:21
e=1/4c + 1/8o + 1/4e
garretth 2017-02-08 20:53:21
e=1/4c + 1/8o + 1/4e
garretth 2017-02-08 20:53:21
e=1/4c + 1/8o + 1/4e
amwmath 2017-02-08 20:53:21
$\frac14c+\frac14e+\frac18\rm o$\
CrystalEye 2017-02-08 20:53:21
1/4c+1/4e+1/8o
FATRaichu 2017-02-08 20:53:21
1/4e+1/4c+1/8o
challengetogo 2017-02-08 20:53:21
e = e/4 + o/8 + c/4
mathfun5 2017-02-08 20:53:21
e = o/8 + c/4 + e/4
devenware 2017-02-08 20:53:26
A particle at $e$ will move each of the 8 directions with probability $\dfrac18$, so \[e=\frac18(3\cdot0+2\cdot c+2\cdot e+1\cdot o).\]
devenware 2017-02-08 20:53:28
Clearing the denominator gives \[8e=2c+2e+o.\]
devenware 2017-02-08 20:53:35
And an expression for $c$?
mathfun5 2017-02-08 20:54:51
c = 1/8 + e/4 + o/8
tdeng 2017-02-08 20:54:51
c=1/8+1/4e+1/8o
amwmath 2017-02-08 20:54:51
$\frac18+\frac14e+\frac18\bar{\rm o}$
mathfun5 2017-02-08 20:54:51
c = 1/8 + e/4 + o/8
FATRaichu 2017-02-08 20:54:51
1/8+1/8o+1/4e
LearnAMC 2017-02-08 20:54:51
c=1/8+1/4e+1/8o
CrystalEye 2017-02-08 20:54:51
1/8+1/4e+1/8o
mathcount2002 2017-02-08 20:54:51
c=1/8+e/4+o/8
devenware 2017-02-08 20:55:05
A particle at $c$ will move each of the 8 directions with probability $\dfrac18$, so \[c=\frac18(1\cdot1+4\cdot0+2\cdot e+1\cdot o).\]
devenware 2017-02-08 20:55:07
Clearing the denominator gives \[8c=1+2e+o.\]
wiler5002 2017-02-08 20:55:42
3 equations and 3 variables so just solve for o
mathchampion1 2017-02-08 20:55:42
so we have a system of equations
devenware 2017-02-08 20:55:47
We are given the system of equations
\begin{align*}
2o&=c+e\\
8e&=2c+2e+o\\
8c&=1+2e+o\\
\end{align*}
and we want to find $o$.
devenware 2017-02-08 20:55:48
What should we do first?
ToneLoke 2017-02-08 20:56:38
substitute o into the bottom two
abvenkgoo 2017-02-08 20:56:38
subtract the bottom two equations
mathfever 2017-02-08 20:56:38
subtract the 3rd equation from the 2nd
devenware 2017-02-08 20:56:43
Funny enough, getting rid of $o$ is a decent first step, but do you see a way to simplify the above equations first, so that maybe we don't have to?
wiler5002 2017-02-08 20:57:50
sub o for c+e
stronto 2017-02-08 20:57:50
replace 2c+2e with 4o
Octophi 2017-02-08 20:57:50
sub in $2c+2e=4o$ into second
shakeNbake 2017-02-08 20:57:50
plug c + e = 2o into the 2nd equation
devenware 2017-02-08 20:57:55
In the second equation, we see $c+e$, but we know that's $2o$! Therefore\[8e=2(c+e)+o=5o.\]
devenware 2017-02-08 20:57:57
\begin{align*}
2o&=c+e\\
8e&=5o\\
8c&=1+2e+o\\
\end{align*}
devenware 2017-02-08 20:58:03
Now we have $e$ in terms of $o$. What is $c$ in terms of $o$?
xiongxiong 2017-02-08 20:59:29
11/8 o
CrystalEye 2017-02-08 20:59:29
11/8o
stronto 2017-02-08 20:59:29
11o/8
bobjoe123 2017-02-08 20:59:29
11o/8 = c
bobjoe123 2017-02-08 20:59:29
$\frac{11o}{8} = c$
devenware 2017-02-08 20:59:34
Combining the first two equations gives us \[2o=c+\frac58o,\] so \[c=\frac{11}8o.\]
devenware 2017-02-08 20:59:36
This makes sense: you're more likely to win from $c$ than from $e$, and $o$ is the average of the two.
devenware 2017-02-08 20:59:38
Now we can use the last equation to find $o$. What is it?
xiongxiong 2017-02-08 21:00:53
4/35
Archos 2017-02-08 21:00:53
4/35
FATRaichu 2017-02-08 21:00:53
4/35
Acstar00 2017-02-08 21:00:53
4/35
bobjoe123 2017-02-08 21:00:53
4/35
brainiac1 2017-02-08 21:00:53
$o = \frac{4}{35}$
ToneLoke 2017-02-08 21:00:53
4/35
devenware 2017-02-08 21:00:57
Substituting gives\[11o=1+\frac54o+o,\]so \[44o=4+9o.\] Therefore $o=\dfrac{4}{35}$.
devenware 2017-02-08 21:00:57
And the answer?
CrystalEye 2017-02-08 21:01:50
39
mathfun5 2017-02-08 21:01:50
39
LearnAMC 2017-02-08 21:01:50
E
mathchampion1 2017-02-08 21:01:50
39
garretth 2017-02-08 21:01:50
39
Acstar00 2017-02-08 21:01:50
(E) 39
xiongxiong 2017-02-08 21:01:50
E
garretth 2017-02-08 21:01:50
woops 39
FATRaichu 2017-02-08 21:01:50
39, E
challengetogo 2017-02-08 21:01:50
39!
amwmath 2017-02-08 21:01:50
39
First 2017-02-08 21:01:50
E
goodball 2017-02-08 21:01:50
39
AnaT129 2017-02-08 21:01:50
E-39!!!!!
mathmagician 2017-02-08 21:01:50
39 of course!
ToneLoke 2017-02-08 21:01:50
E
brainiac1 2017-02-08 21:01:50
4+35=39
bobjoe123 2017-02-08 21:01:50
EEEEE
mathfun5 2017-02-08 21:01:50
(E) 39!
PenguinJoe 2017-02-08 21:01:50
39
WW92030 2017-02-08 21:01:50
E
devenware 2017-02-08 21:01:55
The answer is $4+35=\boxed{39}$, (E).
RemiaFlower 2017-02-08 21:02:11
WE DID IT!!!
devenware 2017-02-08 21:02:24
Awesome!
devenware 2017-02-08 21:02:33
AMC 12 Problem 23 was a repeat of Problem 24 on the AMC 10, so we'll skip it now.
devenware 2017-02-08 21:02:36
That means. . .
devenware 2017-02-08 21:02:43
24. Quadrilateral $ABCD$ is inscribed in circle $O$ and has sides $AB=3$, $BC=2$, $CD = 6$, and $DA = 8$. Let $X$ and $Y$ be points on $\overline{BD}$ such that \[\frac{DX}{BD} = \frac14\qquad\text{and}\qquad\frac{BY}{BD} = \frac{11}{36}.\]Let $E$ be the intersection of line $AX$ and the line through $Y$ parallel to $\overline{AD}$. Let $F$ be the intersection of line $CX$ and the line through $E$ parallel to $\overline{AC}$. Let $G$ be the point on circle $O$ other than $C$ that lies on $CX$. What is $XF\cdot XG$?
$\phantom{hi}$
$\text{(A) } 17 \quad
\text{(B) } \dfrac{59-5\sqrt{2}}{3} \quad
\text{(C) } \dfrac{91-12\sqrt{2}}{4} \quad
\text{(D) } \dfrac{67-10\sqrt{2}}{3} \quad
\text{(E) } 18$
devenware 2017-02-08 21:02:57
Well 145.5 is a very respectable score. Problem 25 now?
amburger66 2017-02-08 21:03:26
noooo
challengetogo 2017-02-08 21:03:26
never!
LearnAMC 2017-02-08 21:03:26
no
First 2017-02-08 21:03:26
No
RemiaFlower 2017-02-08 21:03:38
NO
mathfun5 2017-02-08 21:03:38
NOOOOO
devenware 2017-02-08 21:03:39
Fine, fine. Then tell me what we should do first?
CrystalEye 2017-02-08 21:04:30
draw a diagram
First 2017-02-08 21:04:30
Draw a diagram
challengetogo 2017-02-08 21:04:30
draw the diagram
kevindk 2017-02-08 21:04:30
diagram
amburger66 2017-02-08 21:04:30
draw a diagram
summitwei 2017-02-08 21:04:30
diagram pl0x
Bill9000 2017-02-08 21:04:30
DIAGRAM
ToneLoke 2017-02-08 21:04:30
something somesthing diagram
Cytrew 2017-02-08 21:04:30
diagram!
brainiac1 2017-02-08 21:04:30
this needs a well-drawn diagram
raxu 2017-02-08 21:04:30
Draw a diagram.
yid 2017-02-08 21:04:30
diagram?
Jayjayliu 2017-02-08 21:04:30
draw something
Cytrew 2017-02-08 21:04:30
draw it!
devenware 2017-02-08 21:04:37
devenware 2017-02-08 21:04:40
OK, that's where we start. Next we construct $E$ by extending $AX$ and intersecting the line through $Y$ parallel to $\overline{AD}$.
devenware 2017-02-08 21:04:41
devenware 2017-02-08 21:04:55
After that we construct $F$ by extending $CX$ and intersecting with the line through $E$ parallel to $\overline{AC}$.
devenware 2017-02-08 21:04:56
devenware 2017-02-08 21:05:06
The point $G$ is the intersection of $CX$ and the circle.
devenware 2017-02-08 21:05:13
Now this is a mess of lines, isn't it? We're trying to find $XF\cdot XG$.
First 2017-02-08 21:05:38
Looks scary
devenware 2017-02-08 21:05:40
Any ideas?
AnaT129 2017-02-08 21:06:12
power of a point or similar triangles?
dr3463 2017-02-08 21:06:12
This is a wild guess but can we use power of a point theorem
brainiac1 2017-02-08 21:06:12
looks like power of a point
pandadude 2017-02-08 21:06:12
power of point!
Cytrew 2017-02-08 21:06:12
maybe a bit like power of a point??
Jayjayliu 2017-02-08 21:06:12
power of a point
goodball 2017-02-08 21:06:12
Power of a point?
wiler5002 2017-02-08 21:06:12
Looks kinda like power of a point
devenware 2017-02-08 21:06:25
Definitely looks like power of a point or similar triangles are going to be relevant here.
devenware 2017-02-08 21:06:50
That is, we want to rewrite $XF \cdot XG$ without using $F.$
devenware 2017-02-08 21:07:04
We can do this with power of a point or similar triangles, I'll try similar triangles.
devenware 2017-02-08 21:07:25
One thing to note toward that goal, though. This problem has an interesting property: the configuration is constructed in a sequence of steps. The last thing we constructed was $F$, so it depends on the things we've constructed before. We should be able to "eliminate" $F$ from the problem by unwinding the construction.
devenware 2017-02-08 21:07:30
Let's highlight what we used to construct $F$:
devenware 2017-02-08 21:07:32
devenware 2017-02-08 21:07:33
Now, what do we have here?
Bill9000 2017-02-08 21:07:54
two similar triangles!
mathfun5 2017-02-08 21:07:54
similar triangles!!!
First 2017-02-08 21:07:54
A pair of similar triangles
shakeNbake 2017-02-08 21:07:54
similar triangles
brainiac1 2017-02-08 21:07:54
two similar triangles
1023ong 2017-02-08 21:07:54
similar triangles
User2013 2017-02-08 21:07:54
similar triangles
challengetogo 2017-02-08 21:07:56
similar triangles!
devenware 2017-02-08 21:07:59
Which triangles?
bobjoe123 2017-02-08 21:08:57
ACX~EFX!!!!!!!!
mathfun5 2017-02-08 21:08:57
XFE and XCA
wiler5002 2017-02-08 21:08:57
FXE and CXA
Bill9000 2017-02-08 21:08:57
ACX and EFX
devenware 2017-02-08 21:09:05
Here we have parallel lines $\overline{FE}\parallel\overline{AC}$, so we get some similar triangles $\triangle FXE\sim\triangle CXA$.
devenware 2017-02-08 21:09:27
That's something to work with. However, we have an $XG$ in here, too...
devenware 2017-02-08 21:09:54
If we let $H$ be the other intersection with the circle, we also get $\triangle FXE\sim\triangle HXG$, as these are triangles formed by intersecting chords.
devenware 2017-02-08 21:09:56
devenware 2017-02-08 21:09:58
So how can we rewrite $XF\cdot XG$?
devenware 2017-02-08 21:11:03
Hint: Use the similarity ratio from what we just got.
LearnAMC 2017-02-08 21:12:03
XE*XH
AnaT129 2017-02-08 21:12:03
XH*XE
brainiac1 2017-02-08 21:12:03
$XH \cdot XE$
devenware 2017-02-08 21:12:06
By the similarity $\triangle FXE\sim\triangle HXG$, we get \[\frac{XF}{XH}=\frac{XE}{XG}.\] Now we can "replace" the line $CF$ with the line $AE$ by writing\[XF\cdot XG=XH\cdot XE.\]
devenware 2017-02-08 21:12:17
Notice that Power of a Point on $X$ is equivalent to invoking $\triangle CXA\sim\triangle HXG$ - that's a different way to visualize the same step.
devenware 2017-02-08 21:12:23
(And it's how most of you wanted to do it.)
dr3463 2017-02-08 21:12:29
but how does that help us?
devenware 2017-02-08 21:12:32
Good question!
devenware 2017-02-08 21:12:35
Now we can completely forget the points $F$ and $G$:
devenware 2017-02-08 21:12:37
brainiac1 2017-02-08 21:12:50
good riddance
devenware 2017-02-08 21:12:52
What should we focus on now?
LearnAMC 2017-02-08 21:13:33
H and E
goodball 2017-02-08 21:13:33
Points H and E
AnaT129 2017-02-08 21:13:33
removing points E and H?
devenware 2017-02-08 21:13:43
Yeah, I don't like $E$ either. Let's try to get rid of it.
devenware 2017-02-08 21:13:48
Let's throw down just those pieces that are relevant to the construction of $E$:
devenware 2017-02-08 21:13:51
devenware 2017-02-08 21:13:54
Let's play the same game again. What triangles are immediately similar given that $\overline{EY}\parallel\overline{AD}$?
wiler5002 2017-02-08 21:15:17
EXY and AXD
LearnAMC 2017-02-08 21:15:17
XYE and XDA
pandadude 2017-02-08 21:15:17
XEY, XAD
brainiac1 2017-02-08 21:15:17
AXD and EXY
bobjoe123 2017-02-08 21:15:17
ADX and EYX
1023ong 2017-02-08 21:15:17
EYX, ADX
challengetogo 2017-02-08 21:15:17
ADX ~ EYX
Ani10 2017-02-08 21:15:17
ADX and EYX
bobjoe123 2017-02-08 21:15:17
ADX and EYX are similar!!!!
ArcticWolf 2017-02-08 21:15:17
AXD and EXY?
devenware 2017-02-08 21:15:22
Since those lines are parallel, we know $\triangle EXY\sim\triangle AXD$.
devenware 2017-02-08 21:15:32
Now we also need $HX$ in here. How do we get that?
devenware 2017-02-08 21:17:04
Any other triangles that are similar to $AXD$?
devenware 2017-02-08 21:17:19
Let me add a little more to the diagram and see if we see anything.
devenware 2017-02-08 21:17:22
adyj 2017-02-08 21:18:13
XHB
wiler5002 2017-02-08 21:18:13
HXB
LearnAMC 2017-02-08 21:18:13
BXH
dr3463 2017-02-08 21:18:13
I mean BHX
Ani10 2017-02-08 21:18:13
ADX similar to BHX!
devenware 2017-02-08 21:18:24
Since $DX$ and $AH$ intersect at $X$, we know $\triangle AXD\sim\triangle BXH$. Thus

\[\triangle EXY\sim\triangle BXH.\]
devenware 2017-02-08 21:18:28
What does that tell us about $XH\cdot XE$?
drazn98 2017-02-08 21:19:44
$\frac{XE}{XY}=\frac{BX}{XH}$
wiler5002 2017-02-08 21:19:44
$XY*BX$
Ani10 2017-02-08 21:19:44
YX*BX
LearnAMC 2017-02-08 21:19:44
XY*XB
devenware 2017-02-08 21:19:49
By similarity,

\[XH\cdot XE = XY\cdot XB.\]
devenware 2017-02-08 21:20:02
Okay, so we can finally ditch $E$.
devenware 2017-02-08 21:20:09
We no longer have anything beyond our original cyclic quadrilateral. Let's delete all the extra nonsense and add in the numbers from the problem:
devenware 2017-02-08 21:20:11
AnaT129 2017-02-08 21:20:20
yesssss the diagram is nice now
devenware 2017-02-08 21:20:24
Agreed.
wiler5002 2017-02-08 21:20:54
find the total length of $BD$?
brainiac1 2017-02-08 21:20:54
Looks like we now have to find BD
devenware 2017-02-08 21:21:00
Well, we have these ratios and they let us write everything in terms of $BD$.
devenware 2017-02-08 21:21:00
\begin{align*}

XF\cdot XG

&=XH\cdot XE\\

&=XY\cdot XB\\

&=(BD-BY-DX)\cdot (BD-DX)\\

&=\left(1-\frac{11}{36}-\frac14\right)BD\cdot \left(1-\frac14\right)BD\\

&=\frac{16}{36}\cdot\frac34\cdot BD^2\\

&=\frac{BD^2}3.

\end{align*}
devenware 2017-02-08 21:21:09
So all we need now is $BD$. In fact, we don't need any of those other numbers. I wonder why they didn't just give us $BD$ to begin with. . .
devenware 2017-02-08 21:21:11
How can we find $BD$?
drazn98 2017-02-08 21:22:01
law of cosines on ABD and CBD
BobaFett101 2017-02-08 21:22:01
law of cosines
Alex2 2017-02-08 21:22:01
Use law of cosines twice, get system of equations and solve
brainiac1 2017-02-08 21:22:01
The law of cosines on the two opposite angles
devenware 2017-02-08 21:22:06
We can use the Law of Cosines. We know that $\angle A$ and $\angle C$ are supplementary, so $\cos A=-\cos C$. Let's call this value $c$.
devenware 2017-02-08 21:22:09
The Law of Cosines on $\triangle DAB$ at angle $A$ gives

\begin{align*}

BD^2&=AD^2+AB^2-AD\cdot AB \cdot c\\

&=8^2+3^2-8\cdot3c\\

&=73-24c.

\end{align*}
devenware 2017-02-08 21:22:11
Likewise, the Law of Cosines on $\triangle DCB$ at angle $C$ gives

\begin{align*}

BD^2&=CD^2+CB^2+CD\cdot CB \cdot c\\

&=6^2+2^2+6\cdot2c\\

&=40+12c.

\end{align*}
devenware 2017-02-08 21:22:17
(You're welcome for the algebra. )
goodball 2017-02-08 21:22:30
Thank you.
pandadude 2017-02-08 21:22:37
Thanks!
devenware 2017-02-08 21:22:39
devenware 2017-02-08 21:22:55
So we're left with

\begin{align*}

BD^2&=73-24c\\

BD^2&=40+12c.

\end{align*}
devenware 2017-02-08 21:23:09
If we double the second and add we get

\[3BD^2=73+2\cdot40=153,\]so $BD=\sqrt{51}.$
devenware 2017-02-08 21:23:12
What's the final answer?
bobjoe123 2017-02-08 21:24:04
17
goodball 2017-02-08 21:24:04
17
wiler5002 2017-02-08 21:24:04
A
First 2017-02-08 21:24:04
A
CharlesHong 2017-02-08 21:24:04
A
LearnAMC 2017-02-08 21:24:04
A
goodball 2017-02-08 21:24:04
A
FATRaichu 2017-02-08 21:24:04
17
bobjoe123 2017-02-08 21:24:04
!7!!!!!!!!!!17!!!!!!!!
dr3463 2017-02-08 21:24:04
17
AnaT129 2017-02-08 21:24:04
17 - A
challengetogo 2017-02-08 21:24:04
A
brainiac1 2017-02-08 21:24:04
17
mathfun5 2017-02-08 21:24:04
51/3 = 17
summitwei 2017-02-08 21:24:04
17
mathfun5 2017-02-08 21:24:04
17!!!
devenware 2017-02-08 21:24:09
We want \[XF\cdot XG=\frac{BD^2}3=17,\] so the answer is (A), $\boxed{17}$.
RemiaFlower 2017-02-08 21:24:19
17!!
devenware 2017-02-08 21:24:26
17!!!
RemiaFlower 2017-02-08 21:24:38
YAY we did it again!!!!
dr3463 2017-02-08 21:24:38
We did it!!
Cytrew 2017-02-08 21:24:38
YAY
goodball 2017-02-08 21:24:38
YAY,we finally did it!!!!!!!!!!!!!
brainiac1 2017-02-08 21:24:38
One problem left
adyj 2017-02-08 21:24:38
Let's finish this up.
devenware 2017-02-08 21:24:46
Only one to go! Let's do this.
devenware 2017-02-08 21:25:00
Everyone have their probleming scarves on?
Lah-aops 2017-02-08 21:25:19
sure
Bill9000 2017-02-08 21:25:19
Yesss
goodball 2017-02-08 21:25:19
YESSSS!!!!!!!
RemiaFlower 2017-02-08 21:25:19
Yes, and my matching thinking cap ;)
Makorn 2017-02-08 21:25:19
I actually got a science bowl infinity scarf this weekend ;)
challengetogo 2017-02-08 21:25:24
nope but I have my probleming hat on!
summitwei 2017-02-08 21:25:25
lemme get mine
devenware 2017-02-08 21:25:30
Okay cool.
devenware 2017-02-08 21:25:40
Any probleming gear is fine. I often use probleming socks instead.
devenware 2017-02-08 21:26:28
25. The vertices $V$ of a centrally symmetric hexagon in the complex plane are given by \[ V = \left\{ \sqrt{2}i,-\sqrt{2}i,\frac{1}{\sqrt{8}}(1+i),\frac{1}{\sqrt{8}}(-1+i),\frac{1}{\sqrt{8}}(1-i), \frac{1}{\sqrt{8}}(-1-i)\right\}.\]For each $j$, $1\le j\le12$, an element $z_j$ is chosen from $V$ at random, independently of the other choices. Let $P = \prod_{j=1}^{12}z_j$ be the product of the 12 numbers selected. What is the probability that $P=-1$?
$\phantom{hi}$
$\text{(A) } \dfrac{5\cdot 11}{3^{10}} \quad
\text{(B) } \dfrac{5^2\cdot 11}{2\cdot 3^{10}} \quad
\text{(C) } \dfrac{5\cdot 11}{3^9} \quad
\text{(D) } \dfrac{5\cdot 7\cdot 11}{2\cdot 3^{10}} \quad
\text{(E) } \dfrac{2^2 \cdot 5\cdot 11}{3^{10}}$
devenware 2017-02-08 21:26:54
OK, I see $1+i$. What should you always think when you see $1+i$?
Jayjayliu 2017-02-08 21:27:50
45 degrees
Jayjayliu 2017-02-08 21:27:50
(1+i)^4=-4
BobaFett101 2017-02-08 21:27:50
e^(i pi/4)
devenware 2017-02-08 21:28:03
The complex number $1+i$ should remind you of eighth roots of unity. That is, $(1+i)^2 = 1+2i-1 = 2i$, so \[(1+i)^8=(2i)^4=2^4i^4=2^4.\] Dividing by $\sqrt2$ gives\[\left(\frac{1+i}{\sqrt2}\right)^8=1.\]
devenware 2017-02-08 21:28:20
Let's write $\omega=\dfrac{1+i}{\sqrt2}$. Can we rewrite $V$ in terms of $\omega$?
devenware 2017-02-08 21:29:13
What is $\sqrt{2}i$ in terms of $\omega$?
devenware 2017-02-08 21:29:32
Okay, how about this, what is $i$ in terms of $\omega$?
devenware 2017-02-08 21:30:04
(Think $\omega$ is an 8th root of unity -- what about $i$?)
mathman3880 2017-02-08 21:30:33
\omega^2
amburger66 2017-02-08 21:30:33
w^2
wiler5002 2017-02-08 21:30:33
%\omega^2$
BobaFett101 2017-02-08 21:30:33
w^2
xingxia_alexye 2017-02-08 21:30:33
w^2
devenware 2017-02-08 21:30:49
Right, $i$ is $\omega^2$.
devenware 2017-02-08 21:30:55
So $\sqrt{2}i$ is?
brainiac1 2017-02-08 21:31:30
$\sqrt{2} \omega^2$
amwmath 2017-02-08 21:31:30
$\omega^2\sqrt2$
mathfun5 2017-02-08 21:31:30
sqrt(2)*w^2
wiler5002 2017-02-08 21:31:30
$\sqrt2\omega^2$
mathman3880 2017-02-08 21:31:30
$sqrt{2}*\omega^2$
First 2017-02-08 21:31:30
$\sqrt{2} w^2$
devenware 2017-02-08 21:31:46
Great. One down, 5 more to go.
devenware 2017-02-08 21:31:55
What is $-\sqrt{2}i$ in terms of $\omega$?
mathfun5 2017-02-08 21:32:17
-sqrt(2)*w^2
wiler5002 2017-02-08 21:32:17
$-\sqrt2\omega^2$
xingxia_alexye 2017-02-08 21:32:17
-sqrt(2)w^2
challengetogo 2017-02-08 21:32:17
-sqrt(2)*w^2
devenware 2017-02-08 21:32:23
True, but can we write that without any negatives?
drazn98 2017-02-08 21:33:34
$sqrt{2}{\omega}^{6}$
wiler5002 2017-02-08 21:33:34
$\sqrt2\omega^6$
brainiac1 2017-02-08 21:33:34
$\omega^6 \sqrt{2}$
amwmath 2017-02-08 21:33:34
$\sqrt2\,\omega^6$
BobaFett101 2017-02-08 21:33:34
rt(2)w^6
devenware 2017-02-08 21:33:35
Right, $-\omega^2 = -i = \omega^6$.
devenware 2017-02-08 21:33:52
So $-\sqrt{2}i$ is $\sqrt{2}\omega^6$.
devenware 2017-02-08 21:34:20
Okay, the rest are a little easier. Can someone give them to me?
drazn98 2017-02-08 21:35:04
$frac{\omega}{2}, \frac{{\omega}^3}{2}, frac{{\omega}^5}{2}, \frac{{\omega}^7}{2}$
pandadude 2017-02-08 21:35:04
w/2,w^3/2,w^5/2,w^7/2
Sparks29032 2017-02-08 21:35:04
the third one is w/2
devenware 2017-02-08 21:35:12
Right. Here's the whole set written out:
devenware 2017-02-08 21:35:18
\[V=\left\{

\sqrt2\omega^2,\sqrt2\omega^6,\frac{\omega}2,\frac{\omega^3}2,\frac{\omega^5}2,\frac{\omega^7}2

\right\}.\]
goodball 2017-02-08 21:35:34
Nice.
devenware 2017-02-08 21:35:40
Yeah, that looks a lot cleaner huh?
warrenwangtennis 2017-02-08 21:36:03
why did we not just write them in polar form?
devenware 2017-02-08 21:36:15
I chose to write this in this way since that's how I solved the problem and it's also good to try to become fluent with roots of unity.
devenware 2017-02-08 21:36:21
It probably helps to see these in the complex plane with the unit circle:
devenware 2017-02-08 21:36:23
brainiac1 2017-02-08 21:36:31
now do we have to decipher that weird product in the second part?
devenware 2017-02-08 21:36:33
How many total ways can we construct the product $P$?
warrenwangtennis 2017-02-08 21:37:36
6^12
Jayjayliu 2017-02-08 21:37:36
6^12
mathfun5 2017-02-08 21:37:36
6^12
amwmath 2017-02-08 21:37:36
$6^{12}$
brainiac1 2017-02-08 21:37:36
6^12
Blocry 2017-02-08 21:37:36
6^12
drazn98 2017-02-08 21:37:36
$6^{12}$
yid 2017-02-08 21:37:38
6^12?
devenware 2017-02-08 21:37:44
There are $6^{12}$ total possible strings

\[P=z_1z_2\cdots z_{12}.\]
devenware 2017-02-08 21:37:54
Okay, there's our denominator. Unfortunately, every denominator in the answer choices for the problem is a factor of this, so we're no closer to solving the problem.
devenware 2017-02-08 21:38:22
The product has to have the form $2^a\omega^b$. We want to find all the products that evaluate to $2^0\omega^4$.
devenware 2017-02-08 21:38:35
Does the $2^0$ tell us anything interesting?
pandadude 2017-02-08 21:39:28
so we need 8 of the first 2 and 4 of the last 4 so we can get a magnitude of 1
warrenwangtennis 2017-02-08 21:39:28
take 8 $sqrt2$s and 4 1/2's
15Pandabears 2017-02-08 21:39:28
You need 8 of the sqrt2 and 4 of the 1/2
drazn98 2017-02-08 21:39:28
the magnitude of the product has to be 1, i.e. pick 8 $\sqrt{2}$'s and 4 $\frac{1}{2}$
wiler5002 2017-02-08 21:39:28
have to choose 8 from the first 2 and 4 from the last 4
summitwei 2017-02-08 21:39:28
# of sqrt(2) is twice # of 1/2
devenware 2017-02-08 21:39:36
In order to get a modulus of 1, we need to pick 8 "big" elements from

\[B=\left\{

\sqrt2\omega^2,\sqrt2\omega^6

\right\}

\] and four "little" elements from

\[

L=\left\{

\frac{\omega}2,\frac{\omega^3}2,\frac{\omega^5}2,\frac{\omega^7}2

\right\}.

\]
devenware 2017-02-08 21:39:42
How many total products can we construct like that (ignoring the argument for now)?
devenware 2017-02-08 21:40:32
Let's think about this. We have to have a product that looks like this:
devenware 2017-02-08 21:40:39
\[bb\ell b\ell bbb\ell \ell bb\]
devenware 2017-02-08 21:40:45
With 8 b's and 4 l's.
devenware 2017-02-08 21:41:03
In how many ways can we place 8 b's and 4 l's?
warrenwangtennis 2017-02-08 21:41:42
12 choose 4
mathman3880 2017-02-08 21:41:42
12C4
brainiac1 2017-02-08 21:41:42
12 choose 4 or 495
Sparks29032 2017-02-08 21:41:42
495?
Jayjayliu 2017-02-08 21:41:43
12 choose 4
xingxia_alexye 2017-02-08 21:42:01
C(12,4)
goodball 2017-02-08 21:42:01
12 choose4?
devenware 2017-02-08 21:42:03
There are 12 spaces in total, and we need to choose 4 of them to be $\ell$, so there are $\binom{12}{4}$ ways to do this.
devenware 2017-02-08 21:42:26
Once those are placed, in how many ways can we replace the $b$'s with elements from $B$?
mathman3880 2017-02-08 21:43:44
256
15Pandabears 2017-02-08 21:43:44
2^8
amwmath 2017-02-08 21:43:44
$2^8$
goodball 2017-02-08 21:43:44
256?
wiler5002 2017-02-08 21:43:44
$2^8$
ScienceSpirit 2017-02-08 21:43:46
2^8
devenware 2017-02-08 21:43:53
There are $2^8 = 256$ ways to do that.
devenware 2017-02-08 21:44:02
In how many ways can we replace the $\ell$'s with elements from $L$?
warrenwangtennis 2017-02-08 21:44:35
$4^4$
xingxia_alexye 2017-02-08 21:44:35
2^8
pandadude 2017-02-08 21:44:35
256
ScienceSpirit 2017-02-08 21:44:35
4^4
brainiac1 2017-02-08 21:44:35
4^4=256
devenware 2017-02-08 21:44:43
There are $4^4 = 256$ ways to do that.
devenware 2017-02-08 21:44:49
Therefore, ignoring the argument, we have \[N=\binom{12}82^84^4=\frac{12\cdot11\cdot10\cdot9}{24}\cdot2^{16}=2^{16}3^2\cdot5\cdot11\]strings that have the right modulus.
goodball 2017-02-08 21:45:25
Including the argument?
devenware 2017-02-08 21:45:29
What are all the possible values of our product if it has modulus 1?
devenware 2017-02-08 21:47:06
Okay hold on.
devenware 2017-02-08 21:47:16
What can we get from the product of 8 elements in $B$?
devenware 2017-02-08 21:47:27
What power of $\omega$ will we always get?
mathman3880 2017-02-08 21:48:35
0mod4?
brainiac1 2017-02-08 21:48:35
a multiple of 4
BobaFett101 2017-02-08 21:48:35
multiple of 4
devenware 2017-02-08 21:48:50
A multiple of 4 definitely.
devenware 2017-02-08 21:49:04
Let's actually write this out and see what we find.
devenware 2017-02-08 21:49:19
Eight copies of elements of $B$ gives us:\[(\pm\sqrt2\omega^2)(\pm\sqrt2\omega^2)(\pm\sqrt2\omega^2)(\pm\sqrt2\omega^2)(\pm\sqrt2\omega^2)(\pm\sqrt2\omega^2)(\pm\sqrt2\omega^2)(\pm\sqrt2\omega^2)=\pm16\omega^{16}.\]
devenware 2017-02-08 21:49:24
How does that simplify?
warrenwangtennis 2017-02-08 21:50:02
plus or minus 16
Jayjayliu 2017-02-08 21:50:02
plusminus 16
brainiac1 2017-02-08 21:50:02
just $\pm 16$
BobaFett101 2017-02-08 21:50:02
w^16 = 1
mathman3880 2017-02-08 21:50:02
+-16
devenware 2017-02-08 21:50:19
Since $\omega^{16} = 1,$ that's just $\pm 16.$
devenware 2017-02-08 21:50:32
Multiplying 4 copies of elements from $L$ is a little harder. We get

\[

\left(\frac{\omega^a}2\right)

\left(\frac{\omega^b}2\right)

\left(\frac{\omega^c}2\right)

\left(\frac{\omega^d}2\right)

=\frac{\omega^{a+b+c+d}}{16}.

\]
devenware 2017-02-08 21:50:39
What can we say about that exponent on $\omega$?
warrenwangtennis 2017-02-08 21:51:52
even
brainiac1 2017-02-08 21:51:52
its even
mathman3880 2017-02-08 21:51:52
0, 2 mod 4
Jayjayliu 2017-02-08 21:51:52
no, always even
wiler5002 2017-02-08 21:51:52
0 mod 2
ScienceSpirit 2017-02-08 21:51:52
it's even?
devenware 2017-02-08 21:52:01
Thankfully we get the $\dfrac1{16}$ that we wanted. The exponent $a+b+c+d$ is a sum of four odd numbers. That means it is even. In fact, it can be any even residue modulo 8. Specifically, we could multiply

\[

\left(\frac{\omega^a}2\right)

\left(\frac{\omega^a}2\right)

\left(\frac{\omega^1}2\right)

\left(\frac{\omega^7}2\right)

=\frac{\omega^{2a}}{16}.

\]
pandadude 2017-02-08 21:52:50
1,-1,i,-i
devenware 2017-02-08 21:52:54
Therefore in our set of products with the right modulus, we always get $P$ equal to $1$, $i$, $-1$, or $-i$.
devenware 2017-02-08 21:53:06
Here's a product equal to 1:
devenware 2017-02-08 21:53:07
\[

(\sqrt2\omega^2)

(\sqrt2\omega^4)

(\sqrt2\omega^4)

(\sqrt2\omega^2)

(\sqrt2\omega^4)

(\sqrt2\omega^2)

(\sqrt2\omega^4)

(\sqrt2\omega^2)

\left(\frac{\omega^1}2\right)

\left(\frac{\omega^7}2\right)

\left(\frac{\omega^1}2\right)

\left(\frac{\omega^7}2\right)

\]
devenware 2017-02-08 21:53:10
Here's a product equal to i:
devenware 2017-02-08 21:53:11
\[

(\sqrt2\omega^2)

(\sqrt2\omega^4)

(\sqrt2\omega^4)

(\sqrt2\omega^2)

(\sqrt2\omega^4)

(\sqrt2\omega^2)

(\sqrt2\omega^4)

(\sqrt2\omega^2)

\left(\frac{\omega^3}2\right)

\left(\frac{\omega^7}2\right)

\left(\frac{\omega^1}2\right)

\left(\frac{\omega^7}2\right)

\]
devenware 2017-02-08 21:53:22
[Yikes those are long]
devenware 2017-02-08 21:53:32
This is a weird question, but which of the four products, $1$, $i$, $-1$, or $-i$, are we most likely to get?
wiler5002 2017-02-08 21:54:07
symmetric
wiler5002 2017-02-08 21:54:07
all equally likely
warrenwangtennis 2017-02-08 21:54:07
all equally likely
letsgomath 2017-02-08 21:54:07
none, they all have probability 1/4
brainiac1 2017-02-08 21:54:07
all of them?
goodball 2017-02-08 21:54:34
Equal chance
devenware 2017-02-08 21:54:39
We can turn a product that's equal to 1 into a product that's equal to $i$ by increasing the exponent on the first $\omega^a/2$ by 2 (like the pair of examples above).
devenware 2017-02-08 21:54:41
That's a bijection! There are the same number of strings that equal 1 as equal $i$. So?
warrenwangtennis 2017-02-08 21:55:33
multiply the total number of ways by 1/4
BobaFett101 2017-02-08 21:55:33
divide by 4
devenware 2017-02-08 21:55:48
So the number of products that are equal to $-1$ is $\dfrac N4$.
devenware 2017-02-08 21:55:50
What's the final answer?
xingxia_alexye 2017-02-08 21:56:41
E
summitwei 2017-02-08 21:56:41
e
warrenwangtennis 2017-02-08 21:56:41
E
wiler5002 2017-02-08 21:56:41
E
devenware 2017-02-08 21:56:46
The probability of getting a product of $-1$ is \[\frac{\frac{N}4}{6^{12}}=\frac{2^{16}\cdot3^2\cdot5\cdot11}{4\cdot6^{12}}=\frac{2^{16}\cdot3^2\cdot5\cdot11}{2^{14}\cdot3^{12}}=\boxed{\dfrac{2^2\cdot5\cdot11}{3^{10}}}.\]
devenware 2017-02-08 21:56:47
The answer is (E).
devenware 2017-02-08 21:56:58
Please join us again on Thursday, February 16, when we will discuss the AMC 10B/12B contests and also again on March 9 and 24 when we will be discussing the AIME I and II contests.
devenware 2017-02-08 21:57:08
Oh, and soon you will be able to find videos for the last 5 problems of each exam on the videos page:
http://www.artofproblemsolving.com/Videos/index.php?type=amc
We chose a few different tactics in their videos than we used today, so it might be worth checking them out.
Cytrew 2017-02-08 21:57:13
yay
goodball 2017-02-08 21:57:13
Yay!!!!!
devenware 2017-02-08 21:57:20
YAY!! WE'RE DONE!
amburger66 2017-02-08 21:57:25
WOW thanks!!
challengetogo 2017-02-08 21:57:25
Thank you!!
dragonfly 2017-02-08 21:57:25
ty
devenware 2017-02-08 21:57:32
THANK YOU FOR COMING!
wiler5002 2017-02-08 21:57:37
THANKS DEVEN
iks92 2017-02-08 21:57:37
Thank you!!
amwmath 2017-02-08 21:57:44
$\huge\color{red}{\heartsuit}$
letsgomath 2017-02-08 21:57:49
When will the transcript of this math jam be available?
devenware 2017-02-08 21:58:03
Yes, you will be able to find it here: https://artofproblemsolving.com/school/mathjams-transcripts in a little bit.
devenware 2017-02-08 21:58:27
Have a nice night everyone!

Copyright © 2024 AoPS Incorporated. This page is copyrighted material. You can view and print this page for your own use, but you cannot share the contents of this file with others.